Kaplan GRE & GMAT Exams Writing Workbook, 3rd Edition (2008)

Chapter 3. Writing Strong Sentences: Grammar, Mechanics, and Style

Strong sentences are the building blocks of effective and top-scoring essays. This chapter takes a detailed look at the nuts and bolts of sentences: the most important things you need to know (or review) to help you earn a top score on the Analytical Writing section. Our Eight Maxims of Effective Sentences cover the essentials of grammar and usage, mechanics, and style, including everything from subject-verb agreement to semicolons, from parallel structure to point of view. Throughout this chapter, you’ll find specific tips and strategies to help you earn top scores from both the E-rater and the human grader.

Let’s start with some observations based on our research.

Bullet In terms of the mechanics of writing, the graders do not set the bar extremely high. A comma splice here and some wordiness there won’t ruin your essay. Your paper absolutely does not have to be perfect to get a 6. However, the fewer excuses you give readers for docking your score, the better. We’ll give you guidelines to help you avoid potentially costly mistakes.

Bullet Many so-called grammar rules are regularly broken by good writers. But the E-rater can’t distinguish between rules that you break for effect and rules that you break because you don’t know better; it simply assumes the latter. We’ll review the rules so you can play it safe.

Bullet It is possible to write in a rather personal, even idiosyncratic, style, and still get a top score. Bear in mind, however, that the E-rater will certainly not be moved by wit, eloquence, or any signs of personality. As for the human grader, after reading hundreds and hundreds of responses to the same question, he or she may be impressed by a well-turned phrase. Or, he or she may be cranky and impatient with anything that looks like posturing. We’ll show you how to write with a style that will boost your score.

EIGHT MAXIMS OF EFFECTIVE SENTENCES

There are dozens and dozens of writing rules and guidelines, and it can be easy to feel overwhelmed by them, especially when you’re under pressure to prepare for an exam. To help you remember what’s most important and feel more in control on test day, we’ve organized those rules and guidelines around eight maxims of effective sentences. Use these eight principles as your guide for the Analytical Writing tasks.

MAXIM 1: BE CORRECT

Both the E-rater and the human reader will expect your writing to follow the conventions of Standard Written English. This section covers the most important rules of grammar, usage, and mechanics as well as the most frequently confused words.

Grammar and Usage

Verb Forms and Tenses

Though a few grammar errors alone won’t necessarily bring down your score, certain types of errors are more likely to strike your human readers as more egregious than others. Verb mistakes are among the most damaging, so here’s a detailed review of common kinds of errors.

Helping. Helping or auxiliary verbs do just that: they help you express exactly when an event will or did take place (e.g., future, past perfect, and conditional tenses). They also convey very specific meanings. Though we often use them interchangeably, may and can actually have two different meanings. May expresses permission while can express-es ability; thus the question “may I?” is actually very different from the question “can I?”

Here’s a list of specific helping verb meanings:

9781419552175_Interior_0028_008

Troublesome Verbs. Time to review those three sets of verbs that tend to give writers so much trouble: lie/layrise/raise, and sit/set.

The key to choosing the right word is to remember which verb in each pair is transitive and which is intransitive. A transitive verb needs an object to receive its action (you can think of it as transferring its action to the object). An intransitive verb does not take an object; it performs the action on itself.

9781419552175_Interior_0029_001

A pneumonic trick to remember which verbs are intransitive: the first vowel in each word is i.

After hours of negotiating, Barker had to lie down and take a nap.
After hours of negotiating, Barker lay his head upon the table and fell asleep.

Nolan likes to rise at 4:30 so he can work for several hours before breakfast.
The Federal Reserve raised the interest rate for the third time this quarter.

Justine is going to sit in the back of the room and observe the training class.
I’ve set everything you’ll need for the conference call on your desk.

Subjunctive and Conditional. The subjunctive voice is actually a simple form but one that has been slipping out of everyday usage over the last several decades. However, it is still required for grammatically correct sentences. The subjunctive is primarily used to express something that is wished for or contrary to fact. To form the subjunctive, use the base form of the verb for all persons and numbers. The only exception is also the most frequently used subjunctive verb, to be. The subjunctive of to be is were:

If I were in your position, I would ask for a second opinion.
The mayor wishes [or wished] that the commercial tax base werelarger.

Phrases such as it is (or any other tense of the verb to be) necessary that... also take the subjunctive form. Other adjectives and adjectival phrases that may be used in this expression include vital, essential, indispensable, preferable, advisable, just as well, better, and best.

CORRECT: It is essential that the President recognize the limits of his mandate.

CORRECT: Is it really necessary that we all study the law in order to protect ourselves from nuisance lawsuits?

CORRECT: The CEO suggested that the secretary revise the report before releasing it.

Watch Out for Would!

A conditional sentence does not normally have would in the if clause. The exception is when would means be willing to, as in Ifyou would dry the dishes, I’ll washthem. In most cases, would or would have in the if clause is wrong:

INCORRECT: If the town would have raised taxes, it wouldn’t have gone bankrupt.

CORRECT: If the town had raised taxes, it wouldn’t have gone bankrupt.

INCORRECT: If we would have been more prepared, we would have landed the account.

CORRECT: If we had been more prepared, we would have landed the account.

Practice 1

Find and correct the mistake(s) in the following sentences. Answers are found on Practice 1.

1. Most of us wish that our parents will be better prepared to face retirement.

2. While we may wish that our physical conditioning would be better, few of us are prepared to invest the time and effort in the kind of exercise and diet that might help us achieve it.

3. Does it make sense to wish that we were armed with lethal claws and teeth like other animals?

4. Although it is simple to mouth words of support for the work of others, it is preferable that they are given negative feedback where it applies.

5. Many scientists recommend that the government make an effort to preserve every species.

6. Judging from recent pricing patterns, it is imperative that the American government begins to regulate retail milk prices.

Gerunds and Infinitives. Although gerunds look like verbs because they end in –ing, they are actually nouns:

Kinsley’s organization promotes awareness of global warming.

Infinitives are formed by combining to + the verb base: to warm.

The main mistake people make with gerunds and infinitives is using the wrong from after a conjugated verb. Fortunately, if you are a native speaker of English, you can usually hear right away when this kind of error is made:

INCORRECT: Kinsley hopes promoting awareness of global warming.

CORRECT: Kinsley hopes to promote awareness of global warming.

For some verbs, either a gerund or infinitive will do, though sometimes one sounds a little better than the other:

CORRECT: Kinsley’s goal is to promote awareness of global warming. [best choice] CORRECT: Kinsley’s goal is promoting awareness of global warming. [also correct]

The verbs like, hate, and other words that express preference fall into this either/or category. But many verbs can only take one form. Here are some general guidelines for when to use infinitives and gerunds.

Gerunds should always follow prepositions and the following verbs:

9781419552175_Interior_0031_002

Infinitives generally follow these verbs, even when separated by a noun or pronoun:

advise (I advise you to go to graduate school)

agree (We agree to stop fighting.)

allow (Please allow us to present you with this award for your service to the community.)

ask (They have asked us to write a proposal for developing the Tulman Park area.)

9781419552175_Interior_0031_008

CORRECT: The CEO advised the stockholders not to sell before reading the latest report.

CORRECT: The CEO recommended waiting for the quarterly report before making a decision.

Subject-Verb Agreement

Is it “he go” or “he goes”? Native speakers of English can generally count on their inner ear to pick the right form without any conscious thought. But in long, complex sentences or sentences with indefinite pronouns, even native speakers can sometimes make mistakes.

First, the principle: Subject-verb agreement means that the subject must agree (be equal to) the verb in number. A singular subject (she) must have a singular verb (understands); a plural subject (they) must have a plural verb (understand). Here’s a review of the most common kinds of subject-verb agreement errors.

Indefinite Pronouns.Every, each, everyone, everybody, anyone, and anybody are all grammatically singular, even though they tend to have plural meanings.

INCORRECT: Each of us have completed a review of our department.

CORRECT: Each of us has completed a review of our department.

INCORRECT: Anyone who attempt to change my mind will only be disappointed.

CORRECT: Anyone who attempts to change my mind will only be disappointed.

With these expressions there are often problems of pronoun-antecedent agreement as well as issues of political correctness when you need a singular pronoun to refer to both men and women. You might want to avoid them altogether and use all instead:

INCORRECT: Every one of the delegates has cast their vote.

CORRECT: Every one of the delegates has cast his or her vote.

CORRECT: All of the delegates have cast their votes.

Practice 2

The following sentences contain various problem areas in subject-verb agreement. Of the two choices provided, choose the verb form that matches the subject of the sentence. Answers are found on Practice 2.

1. Many people in New York travel/travels by subway.

2. Workers in New York often commute/commutes a long way to work.

3. Tourists in New York expect/expects to see the subway.

4. The “redbird”subway cars with a red body has/have been a common sight in New York until recently.

5. The streets of large cities such as New York is/are under-cut by complex networks of subway tunnels.

6. Today, the economy of New York and other large cities is/are booming.

7. New Yorkers with a good income is/are less likely to commute by subway.

8. A worker with a long commute does/do not want to spend hours on the subway.

9. A private car, although convenient, pollutes/pollute the air.

10. Parking in one of New York’s many overcrowded garages is/are also a problem.

11. Every day, Joe and Carla rides/ride the subway to work.

12. Every day, Joe or Carla rides/ride the subway to work.

13. Every day, Joe’s sisters or Carla’s sisters rides/ride the subway to work.

14. Every day, Joe’s sisters or Carla rides/ride the subway to work.

15. Every day, Carla or Joe’s sisters from Long Island rides/ride the subway to work.

16. Everyone enjoys/enjoy a summer vacation.

17. Nobody has/have fun when the Cyclones lose a game.

18. Either of the answers is/are valid in response to that question.

19. Each of the students bring/brings a book to class every day.

20. Many is/are obsessed with reality television these days.

Prepositional Distracters. Errors frequently creep in when the grammatical subject of the sentence is singular but includes a prepositional phrase with a plural object. Remember that the object in a prepositional phrase is never the true subject of the sentence. To help you identify this kind of error, read the sentence without the prepositional phrase. Correct the error by changing the verb to match the true subject, by moving the prepositional phrase, or by recasting the sentence so that the object of the preposition becomes the true subject. (The prepositional phrases are bracketed in the first two examples below.)

INCORRECT: The aim [of all promotional strategies] are to influence the shape of the demand curve.

CORRECT: The aim [of all promotional strategies] is to influence the shape of the demand curve.

CORRECT: All promotional strategies have the goal of influencing the shape of the demand curve.

CORRECT: All promotional strategies aim to influence the shape of the demand curve.

When a singular subject is modified by a prepositional phrase that seems to expand the subject, the verb nonetheless agrees only with the true subject.

CORRECT: The President, along with the Vice President and the members of the Cabinet, is the highest security risk in times of crisis.

CORRECT: The President and the Vice President, together with the members of the Cabinet, are protected by numerous security officers.

Other common expressions that function in a similar way are besides, as well as, inaddition to, and not to mention.

Practice 3

Find and correct the mistake(s) in the following sentences. Answers are found Practice 3.

1. The installation of video cameras in public areas certainly add a measure of security but may eventually erode our right to privacy.

2. Competition for grades, jobs, and mates ultimately benefit society.

3. The main flaw in most of these arguments are the reliance upon unsupported inferences.

A Note about British English

Although ETS graders are not supposed to penalize test takers for applying British grammatical norms, it is best to stick to American norms in instances where British and American grammar differ. In British English, for example, collective nouns are plural.

CORRECT (U.S.): Air Novitrans has a fleet of twelve aging aircraft.

CORRECT (UK): Air Novitrans have a fleet of twelve aging aircraft.

We would advise test-takers to play it safe by following American conventions:

RISKY: A group of professors representing a dozen of our top universities have testified that tenure is an outmoded and counterproductive academic custom.

SAFE: A group of professors representing a dozen of our top universities has testified that tenure is an outmoded and counterproductive academic custom.

Other Interrupters Subject-verb agreement errors are also common when the subject and the verb are separated by an intervening adverbial or adjectival phrase.

INCORRECT: At the national level, each government, regardless of the prevailing economic and political institutions, formulate policies that regulate the international marketing efforts of both domestic and foreign firms.

CORRECT: At the national level, each government, regardless of the prevailing economic and political institutions, formulates policies that regulate the international marketing efforts of both domestic and foreign firms.

INCORRECT: The political cartoon depicting the prime minister as a cowboy corralling immigrants have caused a great deal of controversy.

CORRECT: The political cartoon depicting the prime minister as a cowboy corralling immigrants has caused a great deal of controversy.

Group Nouns. Group or collective nouns refer to a class or group and are almost always treated as singular because they emphasize the group as a single unit or entity. When the context of the sentence makes it clear that the writer is referring to the individual members of the group, then the noun is treated as plural. Group nouns include audience, class, committee, company, family, firm, government, group, jury, team, as well as proper names referring to companies and other corporate entities. (Relevant pronouns and verbs are also underlined in the following examples to emphasize agreement.)

INCORRECT: The research group meet weekly to discuss their progress.

CORRECT: The research group meets weekly to discuss its progress.

CORRECT: The members of the research group meet weekly to discuss their progress.

INCORRECT: The jury is engaged in a heated debate.

CORRECT: The jury are engaged in a heated debate. [Calls attention to the individual members and their opinions.]

CORRECT: The members of the jury are engaged in a heated debate.

Be careful with a number of and the number ofA number of means some, and is plural. The number (whether or not it is followed by a prepositional phrase) is singular.

CORRECT: A number of politicians have urged passage of a Constitutional amendment making same-sex marriage illegal.

CORRECT: The number of politicians who refuse donations from political action groups isextremely small.

Practice 4

Find and correct the mistake(s) in the following sentences. Answers are found on Practice 4.

1. The entire team of scientists were allergic to the very chemicals they were studying.

2. The research team is scheduled to conclude its work this week.

3. A number of Internet companies is doubtless preparing to challenge Google for dominance of the search engine market.

Compound Subjects. First, the easy part: a subject that consists of two or more nouns connected by and takes the plural form of the verb.

CORRECT:The Trash-Site Safety Council, a public interest non-profit organization, and Eco-Farms International an association of organic farmers, are monitoring development of upstate landfill projects.

Either/Neither. Now the trickier part. When the subject consists of two or more nouns connected by or or nor, the verb agrees with the CLOSEST noun.

CORRECT: Either the senators or the President is misinformed.

CORRECT: Either the President or the senators are misinformed.

Practice 5

Find and correct the mistake(s) in the following sentences. Answers are found on Practice 5.

1. The CEO, along with the Board of Directors, are responsible for any infraction of the corporation’s environmental protection policy.

2. Either the Attorney General or his senior assistants has the option of prosecuting such violations.

3. Neither the professor nor the students have any strong interest in postponing the end of classes.

Pronouns

When you answer the telephone and someone asks for you, do you reply “This is she” or “This is her”? Pronouns are marvelously useful in that they save us from having to repeat the names of people and objects over and over. But there are several kinds of pronoun errors that can plague even seasoned writers.

Pronoun-Antecedent Agreement. Though it may sound right to say “Each delinquent client has now paid their bill in full,” the pronoun their is incorrect. The subject is singular: each client. The verb is also singular: has. Thus the pronoun must also be singular to agree with client, its antecedent (the word a pronoun replaces or refers to):

CORRECT: Each delinquent client has now paid its bill in full.

Mistakes often occur when the antecedent is an indefinite pronoun. Singular indefinite pronouns include the following:

9781419552175_Interior_0036_006

These are singular antecedents that require singular pronouns:

INCORRECT: I have asked everyone to write down their preferences for travel accommodations.

CORRECT: I have asked everyone to write down his or her preferences for travel accommodations.

Plural indefinite pronouns include both, few, many, and several. They are plural antecedents that require plural pronouns:

INCORRECT: I have carefully considered your offers, and both are appealing, but itis simply too costly.

CORRECT: I have carefully considered your offers, and both are appealing, but they are simply too costly.

Some indefinite pronouns—all, any, most, none, and some—can be either singular or plural depending upon the noun or pronoun to which the indefinite pronoun refers. Here are some examples (notice also how the verbs changes to agree with the subjects):

SINGULAR: Does any owner still have his or her original deed?

PLURAL: Do any of the owners still have their original deeds?

SINGULAR: None of the waste product can be recycled, so it must be stored in an airtight container and placed in underground storage.

PLURAL: None of the waste products can be recycled, so they must be stored in airtight containers and placed in underground storage.

CORRECT Pronoun Case. So what’s the correct response when someone asks for you on the telephone? Answer: “This is he” or “This is she” or “It is I.” Why? Because in this situation the pronoun is functioning as a subject and must be in the subjective case.

9781419552175_Interior_0037_002

Whenever the pronoun functions as a subject, use the subjective case. Whenever the pronoun functions as an object, use the objective case. Remember that pronouns in prepositional phrases are always objects.

INCORRECT: Wiler promised to keep the content of our conversation between he and I.

CORRECT: Wiler promised to keep the content of our conversation between him and me.

INCORRECT: To who should I address a letter of complaint?

CORRECT: To whom should I address a letter of complaint?

Pronouns in comparative than clauses are always subjects because a verb always follows the pronoun, even if that verb is only implied:

INCORRECT: We have been in business much longer than him.

CORRECT: We have been in business much longer than he

The second version is correct because the sentence includes an unstated but understood verb at the end: We have been in business much longer than he has. Here’s another example:

INCORRECT: The Jensens are more likely to get the contract than us because they have more up-to-date equipment.

CORRECT: The Jensens are more likely to get the contract than we [are] because they have more up-to-date equipment.

Other Pronoun Issues. Finally, two last comments about pronouns.

1. Don’t forget that possessive pronouns do not use an apostrophe. Your is a pronoun; you’re is a contraction of you are.

INCORRECT: When potential customers cancel they’re orders, we need to carefully evaluate they’re reasons for cancellation.

CORRECT: When potential customers cancel their orders, we need to carefully evaluate their reasons for cancellation.

2. The relative pronouns who/whom, that, and which are often misused. Here are the guidelines:

Use who/whom when referring to people.

INCORRECT: Two entrepreneurs which have created multi-million dollar empires and which are under 30 will be the keynote speakers at the seminar.

CORRECT: Two entrepreneurs who have created multi-million dollar empires and who are under 30 will be the keynote speakers at the seminar.

Use that when referring to things.

INCORRECT: ABC Candles offers several candle-making classes which are very popular.

CORRECT: ABC Candles offers several candle-making classes that are very popular.

Use which to introduce clauses that provide information that is not essential to the sentence. Exception: if the clause refers to people, use who.

INCORRECT: ABC Candles, that is located on Elm Street, offers weekly candle-making classes.

CORRECT: ABC Candles, which is located on Elm Street, offers weekly candle-making classes. [The information which is located on Elm Street is not essential to the sentence.]

Modifiers

Modifiers are those words, phrases and clauses that serve to describe (modify) other words in a sentence. Here’s the number one rule regarding modifiers: To keep your sentences clear and correct, always place your modifiers as close as possible to the word(s) they modify.

Misplaced Modifiers. If you violate the proximity rule, you’re likely to end up with a misplaced modifier, which is exactly that—a modifier that is in the wrong place. These are usually quite easily corrected by moving the modifying clause or phrase right next to its subject.

INCORRECT: Satisfied and sleepy after a full bottle of milk, the mother laid her baby in the crib.

CORRECT: The mother laid her baby, satisfied and sleepy after a full bottle of milk, in the crib.

INCORRECT: Frightened by the threat of a bear market, the newspaper reported that investors were apprehensive about buying stocks.

CORRECT: The newspaper reported that investors, frightened by the threat of a bear market, were apprehensive about buying stocks.

Dangling Modifiers. Another common modifier error is the dangling modifier. Here the problem is that the subject of the modifying phrase or clause is different from the subject of the main clause or is simply unclear.

DANGLER: Having studied countless sick cows, they were placed on a diet of organic feed and antibiotics.

[The subject of the introductory participle having studied would have to be scientists, veterinarians, or some other group; but the subject of the main clause, they, refers to the cows themselves.]

UNDANGLED: Having studied countless sick cows, the veterinarians placed the cows on a diet of organic feed and antibiotics.

Practice 6

Find and correct the mistake(s) in the following sentences. Answers are found on Practice 6.

1. Looking at the data carefully, the premises simply will not support the conclusion.

2. Having selected an appropriate brand name, there are still many obstacles to successful marketing of the new product.

3. In this argument, an essential inference is that to pass the course the exam must be taken.

Adjectives and Adverbs. A few short words about adjectives (which modify nouns and pronouns) and adverbs (which modify verbs, adjectives, and other adverbs) to help prevent some common errors:

1. Good is an adjective; well is an adverb.

CORRECT: Irina made several good suggestions for revising the application process.

CORRECT: The new application process is working very well.

2. Use the adjective less to modify singular nouns representing a quantity or degree. Use fewer to modify plural nouns or things that can be counted.

INCORRECT: There are less benefits to outsourcing than we first believed.

CORRECT: There are fewer benefits to outsourcing than we first believed.

3. The comparative form should be used when comparing two things. The superlative should be used when comparing three or more things.

Comparative: -er or more/lessheavier, more innovative

Superlative: -est or most/leastheaviest, most innovative

4. Don’t double up. Only one negative or one comparative is needed.

INCORRECT: So far we have been more luckier than last year with the weather during our sidewalk sales.

CORRECT: So far we have been luckier than last year with the weather during our sidewalk sales.

INCORRECT: We don’t have no doubt that our plan will increase business.

CORRECT: We don’t have any doubt that our plan will increase business.

Parallel Structure

An essential element of proper sentence construction is parallel structure, which is really another variation on the idea that items in a sentence should be balanced. Parallelism means that similar elements in a series, list, or two-part construction (e.g., not only/butalso) should be expressed in parallel grammatical form: all nouns, all infinitives, all gerunds, all prepositional phrases, or all clauses.

INCORRECT: All business students should learn word processing, accounting, and how to program computers.

CORRECT: All business students should learn word processing, accounting, and computer programming.

The parallelism principle applies to any words that might begin each item in a series: prepositions (in, on, by, with, etc.), articles (the, a, an), helping verbs (had, has, would, etc.) and possessives (his, her, our, etc.). Either repeat the word before every element in a series or include it only before the first item. Anything else violates the rules of parallelism. In effect, your treatment of the second element of the series determines the form of all subsequent elements.

INCORRECT: He invested his money in stocks, in real estate, and a home for retired performers.

CORRECT: He invested his money in stocks, in real estate, and in a home for retired performers.

CORRECT: He invested his money in stocks, real estate, and a home for retired performers.

When proofreading, check that each item in the series agrees with the word or phrase that begins the series. In the above example, invested his money is the common phrase that each item shares. You would read “He invested his money in real estate, (investedhis moneyin stocks, and (invested his moneyin a home for retired performers.’’

A number of two-part sentence constructions also call for you to always express ideas in parallel form. These constructions include:

X is as ________________ as Y.
X is more/less ______________ than Y.
The more/less X, the more/less Y.
Both X and Y . . .
Either X or Y . . .
Neither X nor Y . . .
Not only X but also Y. . .

X and Y can stand for as little as one word or as much as a whole clause, but in any case the grammatical structure of X and Y must be identical.

INCORRECT: The downturn in sales was attributed not only to the recession but also because a new competitor entered the market.

CORRECT: The downturn in sales was attributed not only to the recession but also to the fact that a new competitor entered the market.

INCORRECT: Generally, the profits from sales of everyday necessities are not nearly as spectacular as from luxury items.

CORRECT: Generally, the profits from sales of everyday necessities are not nearly as spectacular as those from the sale of luxury items.

It is often rhetorically effective to use a particular construction several times in succession to emphasize a particular idea or series of thoughts. The technique is called parallel construction, and it is effective only when used sparingly because it works on a larger (multiple-sentence) scale. Here’s how parallel construction should be used:

CORRECT PARALLELISM: As a leader, Lincoln inspired a nation to throw off the chains of slavery; as a philosopher, he proclaimed the greatness of the little man; as a human being, he served as a timeless example of humility.

The repetition of the sentence structure (As a X, Lincoln Y…) provides a strong sense of rhythm and organization to the sentence and alerts the reader to yet another aspect of Lincoln’s character. However, careless writers sometimes use a parallel structure for expressions of dissimilar structure.

INCORRECT: They are sturdy, attractive, and cost only a dollar each. [The phrase Theyare makes sense preceding the adjectives sturdy and attractive, but cannot be understood before cost only a dollar each.

CORRECT: They are sturdy and attractive, and they cost only a dollar each.

Practice 7

Find and correct the mistake(s) in the following sentences. Answers are found on Practice 7.

1. For example, I would say that my roommate could be characterized as a poor student because he waited until the last minute to study for exams, wrote his lab reports without completing the assigned experiments, and his motivation was low.

2. It is reasonable for a Kravis Software sales representative to expect that he will have an opportunity to introduce his products at the meeting, that there will be a projector for his slide presentation, and prospective buyers will ask questions about the product.

3. We would like to hire someone who is eager, responsible, and works very hard.

Sentence Fragments

In real life, full-blown grammatically complete sentences are a rarity. Most speech takes the form of what would qualify as a “sentence fragment” if it occurred in writing. But for the Analytical Writing section, you’ll want only complete sentences. Don’t even risk a title: the E-rater will think that’s a fragment, too.

What exactly is a fragment? An incomplete sentence (like this one). To be complete, a sentence must have both a subject and a verb and express a complete thought. Often fragments are dependent clauses that need another sentence to make a complete thought:

FRAGMENT: While we are reorganizing the department. Harrison will be the interim office manager. [The first sentence is a dependent clause that cannot stand alone; while makes it depend upon the second sentence.]

These fragments have an easy fix: simply combine the dependent and independent clauses into one sentence.

COMPLETE THOUGHT: While we are reorganizing the department, Harrison will be the interim office manager.

Other times a fragment is a clause or phrase missing a subject or verb, or both. The solution: add the missing subject or verb, or attach the lose clause or phrase to the core sentence.

FRAGMENT: Cheyenne has been researching ways to conserve energy. Without sacrificing production.

COMPLETE THOUGHT: Cheyenne has been researching ways to conserve energy without sacrificing production.

Sometimes a relative pronoun (that, who, which) causes the fragment, which can be corrected by simply deleting the pronoun.

FRAGMENT: Cheyenne, who has been researching ways to conserve energy without sacrificing production.

COMPLETE THOUGHT: Cheyenne has been researching ways to conserve energy without sacrificing production.

Practice 8

Some of the sentences in the following short passage are sentence fragments. Identify the fragments and fix them by adding or removing the necessary words to make complete sentences. Answers are found on Practice 8.

Everyone feels shy. At some point in life. It is perfectly normal to be concerned about how strangers might view you in an unfamiliar situation. For example, a social gathering or new job. However, shyness can sometimes become a major difficulty. If a person feels overpowering anxiety about common situations such as going to the store, attending classes at school or even walking down the street. That anxiety can interfere with the person’s ability to carry on a normal life. A few shy people develop a serious fear of strangers. Forcing them to restrict or avoid contact with people most of us interact with daily. Such as the mail carrier, co-workers or teachers. These people who feel overwhelmed by the mere thought of contact with a stranger. Such people may have agoraphobia.

9781419552175_Interior_0043_007

Run-on Sentences

Run-ons are sentences that literally run right into each other sometimes there is a comma between them however a comma alone is not strong enough to separate two complete thoughts.

Okay, let’s try that again:

Run-ons are sentences that literally run right into each other. Sometimes there is a comma between them; however, a comma alone is not strong enough to separate two complete thoughts.

There are four ways to correct run-on sentences:

1. Separate the sentences with a period.

INCORRECT: Cheyenne has been researching ways to conserve energy, she will submit her findings by Friday.

CORRECT: Cheyenne has been researching ways to conserve energy. She will submit her findings by Friday.

2. Separate them with a comma and a coordinating conjunction (and, or, for, nor, but, so and yet).

CORRECT: Cheyenne wants to conserve energy, but she is worried about sacrificing production.

3. Separate them with a semicolon. Use this option when the two ideas are closely related.

CORRECT: Cheyenne has been researching ways to conserve energy; she is also seeking more ecofriendly suppliers.

4. Make one sentence dependent upon the other by adding a subordinating conjunction such as since, because, while, although, during, and before.

CORRECT: Although it might mean slightly increased costs, Cheyenne wants to use more eco-friendly suppliers.

It is also correct to separate the two sentences with a dash, but only if you wish to set the second sentence off for emphasis.

CORRECT: Cheyenne wants to conserve energy—but only if it doesn’t sacrifice production.

Note:However is not a subordinating conjunction; it is a conjunctive adverb. Use it with a semicolon, not a comma, to join two independent clauses.

CORRECT: Cheyenne wants to conserve energy; however, she is worried about sacrificing production.

Mechanics

Mechanics refers to the rules for punctuation, capitalization, and spelling. With the E-rater, correct mechanics is especially important, because these are among the easiest mistakes for the program to find.

Punctuation

Comma. If you are one of those writers who finds comma rules confusing, you’re in a bit of luck. In a few cases where a comma used to be required, the comma is now optional:

Bullet before the conjunction (usually and) in a series

Bullet before the coordinating conjunction joining two relatively short independent clauses

Bullet following brief introductory adverbial expression

In all these cases, so long as there is no danger of ambiguity, ETS favors omitting the comma. That doesn’t mean you have to omit it or that you will be penalized if you do; these are still grammar “gray areas.” However, it does mean a little more freedom from comma worries.

That said, let’s review the places where you do need a comma:

71 To set off appositive or non-restrictive material within the sentence—that is, information that is relevant but not essential to the core sentence. (Obviously, if the material occurs at the beginning or the end of the sentence, only one comma will be needed.)

CORRECT: Early retirement was offered to eleven employees, all of whom have been employed for over 25 years.

CORRECT: Dwayne, who has been an employee for over 25 years, has accepted the offer of early retirement.

71 To set off transitional expressions, interjections, or nouns of direct address:

CORRECT: Critics of the proposal, however, will doubtless focus on the unknown risks.

CORRECT: Nevertheless, the proponents of the proposal will surely prevail.

CORRECT: After we review your proposal,Mrs. Jenkins, we’ll let you know our decision.

71To separate direct discourse from the speaker or source (unless the quoted phrase is very short):

CORRECT: I wonder, How can politicians fairly represent such a diverse constituency?

CORRECT: My supervisor informed me, with a straight face, “Your performance is too good for this department. You’re making the rest of us look bad.”

Note that quotation marks are only necessary for direct quotation of an utterance. Otherwise, they are not necessary:

CORRECT: The question is, Do we need more students?

71 Before a coordinating conjunction joining two independent clauses.

CORRECT: It was Kendra’s first time interviewing a potential employee, and she was more nervous than the applicant.

71 After a long introductory phrase or clause.

CORRECT: Against the advice of his accountant, Auggie did not pay estimated taxes.

71Between two modifiers that both modify the same word.

CORRECT: Hani is a perfect example of a self-educated, highly successful entrepreneur. [Both self-educated and highly successful modify entrepreneur.]

71Between items in a series. Again, the comma before the and and last item is optional.

CORRECT: To furnish her new office, Anna bought two filing cabinets, a spacious oak desk, a matching swivel chair, and several framed Ansel Adams photographs. [Remember that the comma before and is optional.]

71 Anywhere else it’s needed to prevent confusion.

CORRECT: When you think about it, it isn’t really all that complicated.

Now, here are some cases where the comma is superfluous:

71 when a direct quotation ends in a question mark or an exclamation point but the sentence continues on, the comma must be omitted:

INCORRECT: “Millions for defense, but not one cent for tribute!,” the newspapers proclaimed.

CORRECT: “Millions for defense, but not one cent for tribute!” the newspapers proclaimed.

71 after very short introductory adverbial expressions if there is no danger of confusion:

INCORRECT: Every winter, the number of fatal traffic accidents increases due to adverse driving conditions.

CORRECT: Every winter the number of fatal traffic accidents increases due to adverse driving conditions.

71 around restrictive (essential) phrases

INCORRECT: Teachers and principals, in the public school system, have been lobbying for more funding for the arts.

CORRECT: Teachers and principals in the public school system have been lobbying for more funding for the arts.

71 before words quoted indirectly

INCORRECT: The Supreme Court has ruled that, no evidence gathered by illegal surveillance methods entered into the trial record.

CORRECT: The Supreme Court has ruled that no evidence gathered by illegal surveillance methods entered into the trial record.

71 after a coordinating conjunction

INCORRECT: Every nation must take measures to protect itself from terrorist attacks, but, the rights of citizens to due privacy and equal protection under the law must be protected.

CORRECT: Every nation must take measures to protect itself from terrorist attacks, but the rights of citizens to due privacy and equal protection under the law must be protected.

Practice 9

Find the mistake(s) in the following sentences. Answers are found on Practice 9.

1. Elementary schools must impart the tools necessary, to teach the basic skills.

2. The role, of providing lifelong assistance to disabled people, belongs to the government which can muster the vast resources needed to properly care for the ill.

3. Advocates of the proposed law however, will most likely insist on the need to forestall improper sharing of intellectual property, and classified information.

4. All the support for this argument is either flawed superfluous or irrelevant.

5. In fact it is just as likely that some other cause can explain why the products at the uptown factory are cheaper to produce.

6. Although discounted the evidence of the second survey is actually more informative to this argument.

Period. How and when to use a period may seem straightforward, but especially when it comes to quotations and abbreviations, there are specific rules you need to follow.

71 An indirect question embedded within a declarative sentence ends with a period, not a question mark.

INCORRECT: We must consider why white-collar crime has risen so rapidly?

CORRECT: We must consider why white-collar crime has risen so rapidly.

71 Only one period should appear at the end of a sentence, even if the sentence ends in an abbreviation or a directly quoted sentence. Likewise, a period cannot be combined with a question mark or exclamation mark.

INCORRECT: Investigators recently reported, “The mysterious pollutants have been traced to the Trenton-based Kwalitee Products, Inc..”.

INCORRECT: Investigators recently reported, “The mysterious pollutants have been traced to the Trenton-based Kwalitee Products, Inc..”

CORRECT: Investigators recently reported, “The mysterious pollutants have been traced to the Trenton-based Kwalitee Products, Inc.”

INCORRECT: Connor’s brief fax consisted of only one word: “Help!”.

CORRECT: Connor’s brief fax consisted of only one word: “Help!”

71 In abbreviations, no space follows internal periods. Where initials are used instead of names, the general practice is to leave internal spaces, although newspapers do not do so. If initials replace a name, the periods and spaces may be omitted. Whichever style you choose, be consistent.

INCORRECT: Many students feel that having the letters “Ph. D.” after their name will make them more competitive on the job market.

CORRECT: Many students feel that having the letters “Ph.D.” after their name will make them more competitive on the job market.

INCORRECT: Since the dissolution of the U. S. S. R., political instability has replaced political repression.

CORRECT: Since the dissolution of the U.S.S.R., political instability has replaced political repression.

71 Periods are omitted from acronyms, words formed from the initial letters of a multi-word name: NATO (North Atlantic Treaty Organization), GOP (Grand Old Party, i.e., Republican Party), CEO (chief executive officer), etc. Many abbreviated company names are treated as acronyms: IBM, AOL, GM.

71 When a quoted phrase appears at the end of a sentence, the period is placed within the quotation marks, even when it did not appear in the original next that is being quoted. (This is the American practice; the British practice is to place the period outside the quotation marks unless it was actually part of the original text.)

AMERICAN: The prevailing ethos of our westward expansion is summed up in the phrase “Manifest Destiny.”

BRITISH: The prevailing ethos of our westward expansion is summed up in the phrase “Manifest Destiny”.

Need we insist? It’s an American test; use the American convention.

Practice 10

Find the mistake(s) in the following sentences. Answers are found on Practice 10.

1. We may well ask ourselves what colleges can possibly do to prevent cheating?

2. Does it actually make sense to ask whether people’s lives are more meaningful now than one hundred years ago?

3. For years, the largest contingents of international peacekeepers have come from the U. K. and the U. S. A..

Semicolon. For the most part, you should use a semicolon only where you could also use a period—it is a punctuation mark that belongs primarily between two independent clauses. The difference between the semicolon and period is that a semicolon still keeps the two clauses together in one sentence. Thus, use a semicolon instead of a period when the two clauses are closely related and you want to keep a connection between them:

PERIOD: Hillary earned her bachelors degree in just two and a half years. She then went on to become a teacher of gifted children. [These two sentences are sequential, but not closely related.]

SEMICOLON: Hillary earned her bachelors degree in just two and a half years; she earned her masters in less than one. [These two sentences are closely related; they both deal with Hillary’s speed in earning her degrees.]

The other time to use a semicolon is when one or more items in a list contain an internal comma. In this case you can avoid confusion by using semicolons instead of commas to separate the elements in the list.

CORRECT: Similar incidents have occurred in Houston, Texas; Nashville, Tennessee; and Davis, California.

Practice 11

Find the mistake(s) in the following sentences. Answers are found on Practice 11.

1. When a society is in agreement on the need to meet social objectives, government generally takes on a powerful role, in such cases, taxation, rather than simply raising money, becomes, in addition, a means of implementing those goals.

2. The assets of such an enterprise might include, for example, $30 million in real estate, equipment and infrastructure, $20 million in cash, investments, and accounts receivable, and $10 million in inventory.

3. Many of the food products marketed by McBurger are notoriously high in cholesterol, fat, and calories; the company’s sales are therefore likely to decline over the next few years if new product lines are not introduced.

Colon. Less frequently used than commas and semicolons, but no less important, is the colon. The colon has three specific functions. Use a colon:

1. To introduce a list of three or more items if the list comes after an independent clause:

INCORRECT: Some of the benefits of the new system include: speed, accuracy, and significant savings.

CORRECT: Some of the benefits of the new system include speed, accuracy, and significant savings

CORRECT: There are three benefits of the new system: speed, accuracy, and significant savings.

Do not use a colon after such as, for example, or including:

INCORRECT: The new system offers many benefits, such as: speed, accuracy, and significant savings.

2. To introduce a quotation if it comes after an independent clause:

INCORRECT: As Thomas Edison said: “Genius is 1% inspiration and 99% perspiration.”

CORRECT: I agree 100% with Thomas Edison: “Genius is 1% inspiration and 99% perspiration.”

3. To introduce an explanation or summary of an independent clause:

CORRECT: There is one essential ingredient of success, and only one: perseverance.

CORRECT: The pitch was a complete failure: the client rejected every item in the proposal.

Hyphen and Dash. Often confused because they look alike, the hyphen (-) and the dash (—) perform very different functions. Use a hyphen to connect two or more words that work together as one object or modifier:

CORRECT: The editor-in-chief occasionally does restaurant reviews.

CORRECT: Our new partnership is clearly a win-win situation.

Use a dash to set off a word, phrase or clause for emphasis:

CORRECT: Pickering—who is just 26 years old—is the youngest CEO of a major corporation.

Question Mark. Use question marks only to indicate a direct question is being asked.

INCORRECT: We often wonder where the time has gone?

CORRECT: We often wonder, where has the time gone?

Do not place a question mark at the end of an indirect question unless the indirect question is itself is embedded in a question.

INCORRECT: A good leader should always ask herself who will follow her lead?

CORRECT: A good leader should always ask herself who will follow her lead.

CORRECT: Should a good leader always ask herself who will follow her lead?

Practice 12

Find the mistake(s) in the following sentences. Answers are found on Practice 12.

1. The prompt poses a simple question: Should the 55 mph limit should be restored?

2. The question posed is whether the 55 mph limit should be restored.

3. Is there an answer to the question of the feasibility of restoring the 55 mph speed limit?

Quotation Marks. Quotation marks enclose text that is a direct quote. They can also be used to indicate a word that is being used as a word.

INCORRECT: This building is zoned for residential office use, the realtor said.

CORRECT: “This building is zoned for residential office use,” the realtor said.

CORRECT: The realtor said this building is zoned for residential office use.

INCORRECT: He used the term proactive five times in his introduction.

CORRECT: He used the term “proactive” five times in his introduction.

Here are a few important notes about quotation marks:

71 In American usage (unlike British), commas and periods should be placed inside quotation marks, even when they are not properly part of the quoted material.

CORRECT: J. D. Salinger’s “Catcher in the Rye,” a novel taught in American high schools for generations, caused a fundamental shift in attitudes toward “dirty language.”

CORRECT: “Ask not what your country can do for you,” urged the President. “Ask what you can do for your country.”

71 Colons and semicolons, on the other hand, should follow the closing quotation mark.

CORRECT: I was not one who believed “my country right or wrong”; in fact, having come to age during the Viet Nam War, I tended to assume that our foreign policy was devoid of moral principle.

71 Question marks and exclamation marks should be placed inside the quotation marks when they are part of the quoted material and outside when they are not. When a question ends with a quotation that is itself a question, there will be only one question mark: the one inside the quotation marks.

INCORRECT: Doesn’t the judge usually ask the jurors, “Have you reached a verdict?”?

CORRECT: Doesn’t the judge usually ask the jurors, “Have you reached a verdict?”

CORRECT: The judge asked the jurors, “Have you reached a verdict?”

71Apostrophe. The apostrophe is used to form most possessives and contractions, in addition to a few plurals (see above). Here are a few reminders to help you avoid apostrophe errors.

71 The apostrophe is used to show possession, not plurality:

INCORRECT: Americans eat million’s of hamburger’s every day.

CORRECT: Americans eat millions of hamburgers every day.

71 When adding a possessive apostrophe to a plural noun ending in s, place the apostrophe after the plural s and omit the possessive s.

INCORRECT: As new products are developed, the manager must devote some thought to those product’s names.

INCORRECT: As new products are developed, the manager must devote some thought to those products’s names.

CORRECT: As new products are developed, the manager must devote some thought to those products’ names.

71 If the plural does not end in s, the possessive is formed, as usual, by adding apostrophe and then s.

INCORRECT: Some peoples’ idea of a sacrifice is not watching TV for one evening.

CORRECT: Some people’s idea of a sacrifice is not watching TV for one evening.

71 If a singular noun ends in s, show possession by adding ‘s.

INCORRECT: Travis relied on Lukas’ expertise to negotiate a fair contract.

INCORRECT: Travis relied on Luka’s expertise to negotiate a fair contract.

CORRECT: Travis relied on Lukas’s expertise to negotiate a fair contract.

71 Remember that possessive pronouns do not take an apostrophe:

INCORRECT: The decision is our’s [your’s, her’s, their’s].

CORRECT: The decision is ours [yours, hers, theirs].

Practice 13

Find the mistake(s) in the following sentences. Answers are found on Practice 13.

1. Three principle’s are at issue in this case.

2. Ultimately, a leader must be guided by her peoples wishes.

3. With very few exceptions, womens’ rights have been adjudicated by the courts over the past few decade’s.

4. Many Prince’s of Wales’ have served long terms as the heir to the crown of Great Britain, and Prince Charles’ case is no exception.

The normal way to change a singular noun to plural is by adding s or es. But when it comes to numbers or other characters, the plural is usually formed by adding ’s:

CORRECT: How many i’s are there in the word Mississippi ?

CORRECT: A valid email address cannot have two @’s.

Decades have traditionally been accorded apostrophes (e.g., 1990’s). These days, however, it is becoming more common to omit the apostrophe (1990s), especially in the abbreviated form, which already uses an apostrophe to indicate the omission of the first two digits of the year:

CORRECT: Having grown up in the 1990s, I believe that world peace is an achievable goal.

UNACCEPTABLE: The ’60’s and ’70’s were years of turbulence in many countries.

ACCEPTABLE: The ‘60s and ‘70s were years of turbulence in many countries.

ACCEPTABLE: The sixties and seventies were years of turbulence in many countries.

Practice 14

Find the mistake(s) in the following sentences. Answers are found on Practice 14.

1. In most developing country’s, up to 80% of the population lives in rural community’s.

2. Republicans and Democrats alike may feel uncomfortable about running on their parties’ platforms.

3. Human’s are not the only primate’s that have been known to murder their own kind: infanticide is well-documented among chimpanzee’s, baboon’s, and several species of monkeys.

Slash. The forward slash (/) has many uses.

It can mean per, as in m3/sec (cubic meters per second).

It can mean and, as in w/d (washer and dryer).

It can indicate abbreviation, as in w/ and w/o (with and without).

In a non-technical academic essay such as the Analytical Writing section, you should avoid these informal or technical uses of the slash.

One other use of the slash is worth avoiding. Apparently, due to the popularization of s/he (read she or he) and other attempts to remedy the lack of a gender-neutral singular pronoun in English, the slash has in recent years been tossed around as a short form of or. This practice is not likely to gain acceptance in professional or academic prose any more than the use of & to replace and. Either spell out or, or let a single option suffice.

UNACCEPTABLE: A successful leader will convince/coerce his/her followers to lay aside their hostilities and focus on the challenges facing the community.

ACCEPTABLE: A successful leader will convince her followers to lay aside their hostilities and focus on the challenges facing the community.

Practice 15

Find the mistake(s) in the following sentences. Answers are found on Practice 15.

1. In the age of electronic media, reading books/magazines is not as important as it once was.

2. Consumers must have the confidence that they have an option to get a refund/exchange any online purchase that does not meet with their satisfaction.

3. Statistics for births/year/person can give a useful measure of a nation’s reproductive status.

Practice 16

The following paragraph contains numerous errors in punctuation. Correct the errors and check your answers against the revised version found on Practice 16.

(1) Abraham Lincoln described the U. S. government as being as he said in the Gettysburg Address “of the people by the people and for the people.” (2) This notion I believe puts the priorities’ between the achievements of institutions, and the achievements of governments, in proper perspective. (3) It is not a question, of the achievements of individuals v. s. the achievements of the government, that individual/ government contrast is a false dilemma. (4) Rather it is the achievements of individuals which make up the achievements of a government that determine the greatness of a nation. (5) This is what Lincoln meant when he referred to the nations “unfinished work:” that each of us has a task to perform and those tasks determine what kind of nation we will live in. (6) With this fundamental argument in mind how can we not join with Lincoln in wishing that such a nation, “shall not perish from the earth”?

Capitalization

Capitalization is not rocket science, but it is easy to lose points for simple errors. These are the most important points to review.

First, the general principle is this: Capitalize proper nouns—the specific names of people, places, and things. Do not capitalize general items.

Specific

General

Souderton High School

my high school

Aunt Bettie

your favorite aunt

A-Plus Computer Services

computer repair company

Now some more specific guidelines. In a GRE or GMAT essay, you must capitalize:

71 the first word in a sentence. Avoid ellipsis points (...): even if you use them correctly, the E-rater is likely to become confused and decide that you have an uncapitalized sentence.

71 people’s names, as well as titles when used with the name (Judge Blackwell) or, in the case of high officials, when used to refer to a particular person.

CORRECT: In the face of enormous resistance to his foreign policy, President Johnson announced that he would not run for re-election in 1968.

CORRECT: The President called a press conference to announce his decision.

CORRECT: The president of the United States is not chosen directly by the people.

71 names of places (but not “the” or “of,” unless they fall at the beginning of a sentence), as well as adjectives derived from place names.

CORRECT: The Library of Congress contains several exhibitions in its online gallery.

CORRECT: There are several exhibitions in the Library of Congress’s online gallery.

71 names of institutions (except, again, “the” or “of” and other minor words, unless they fall at the beginning of a sentence).

CORRECT: I received my letter of acceptance from the University of Pennsylvania today.

CORRECT: The University of Pennsylvania campus is just five miles from my home.

71 special events and periods (names of holidays, historical events, eras), but not centuries.

CORRECT: The exhibit includes drawings and letters from the historic Battle of the Bulge.

71 names of countries, languages, and religions, and adjectives derived from them.

CORRECT: In the tiny country of Bhutan, nearly every citizen is a Buddhist.

71 the stars, planets, and other celestial bodies and structures (the Milky Way, the Crab Nebula); ETS even capitalizes Earth, as well as Moon and Sun when referring to the Earth’s moon and sun.

71 the first and all important words of publication titles, movies, songs, works of art.

CORRECT: The British comedy “The Office,” like the American comic strip “Dilbert,” satirizes life behind the cubicle.

Do NOT capitalize:

71 any word simply for emphasis

INCORRECT: The two essential characteristics of any leader are Decisiveness and Communication Skills.

CORRECT: The two essential characteristics of any leader are decisiveness and communication skills.

71 the names of academic subjects, except when referring to the name of a particular course or to a department.

INCORRECT: I plan to major in Anthropology and minor in French Literature.

CORRECT: I plan to major in anthropology and minor in French literature.

INCORRECT: This semester I am taking anthropology 101 and introduction to french literature.

CORRECT: This semester I am taking Anthropology 101 and Introduction to French Literature.

71 seasons.

71 the first word of a sentence cited in indirect discourse.

INCORRECT: Leah said She was enrolling in the executive MBA program.

CORRECT: Leah said she was enrolling in the executive MBA program.

Practice 17

Find the mistake(s) in the following sentences. Answers are found on Practice 17.

1. During the Middle Ages and the most of the Renaissance, Philosophy was hardly distinct from Theology.

2. By the end of Spring, american and canadian University students have been enjoying their Summer Vacation for well over a month.

3. Widely regarded as the most important U. S. Trade Agreement in the past two decades, the U.S.-Canada Free Trade Agreement phased out all tariffs and quotas between the two countries by the end of the Twentieth Century.

4. Last year, doctor sanford was elected president of the brooklyn council. The president of the council meets regularly with the dean of students.

5. american independence day and memorial day occur during the summer, but labor day is in the fall.

6. The novels of william faulkner are set in the american south, and the author himself lived most of his life south of the mason-dixon line.

Spelling

Frequently Misspelled Words. Most of us could use a good review of these commonly misspelled words—especially those of us who have grown dependent upon the autocorrect feature on our computers.

absence: One a, two e’s.

accommodate, accommodation: Two c’s, two m’s

accompany: Two c’s.

all right: Two words. Alright is NOT all right.

a lot: Always two words, never one; do not confuse with allot.

argument: No e after the u.

calendar:A, e, then another a.

campaign: Remember the aig combination.

cannot: Usually spelled as a single word, except where the meaning is “able not to.”

CORRECT: One cannot ignore the importance of conformity.

CORRECT: Anyone can not pay taxes, but the consequences may be serious.

comparative, comparatively: Yes, comparison has an i after the r. These words don’t.

conscience: Spell it with science.

correspondent, correspondence: No dance.

definite: Spell it with finite, not finate.

develop,development: No e after the p.

embarrass: Two r’s, two s’s.

every day (adv.): Two words with every modifying day. Note that there is also an adjective, everyday, meaning commonplace, usual.

ADVERB: We see this error every day.

ADJECTIVE: Getting stuck behind an elephant in traffic is no longer an everyday occurrence in Katmandu.

exaggerate: One x, two g’s.

foreign: Think of the reign of a foreign king.

grammar: No e.

grateful: Spell it with grate

harass: One r, two s’s.

independent, independence: No dance.

indispensable: It’s something you are not able to dispense with.

judgment: No e on the end of judge.

leisure: Like pleasure but with an i instead of a.

license: In alphabetical order: c then s, not lisence.

maintenance:main, then ten, then ance (reverse alphabetical order for your vowels preceding n).

maneuver: Memorize the unusual eu combo.

no one: Two words. Don’t be mislead by nobodynothingeveryonesomeone, and anyone.

noticeable: Notice that this one keeps the e when adding the suffix.

occur, occurred, occurrence: Double the r when you add a suffix beginning with a vowel.

parallel, unparalleled: Two l’s, then one.

parenthesis (pl. parentheses): Likewise, many other words of Greek origin are spelled with -is in the singular and -es in the plural; among the more common are analysis, diagnosis, prognosis, synthesis, thesis.

perseverance: Only two r’s—sever, not server. Remember that the a in the suffix keeps it from being all e’s.

professor, professional: One f.

pronunciation: Never mind pronounce and pronouncementpronunciation has no o in the second syllable.

questionnaire: Two n’s, one r.

regardless: Not irregardless, an unacceptable yoking of irrespective and regardless.

responsible, responsibility: While the French and Spanish cognates end in –able, it’s –ible in English.

separate: Look for “a rat” in separate.

unanimous:un then an.

vacuum: One c, two u’s.

Practice 18

The following paragraph contains several of the spelling errors described in this section. Locate and correct the commonly misspelled words. There are 37 spelling errors in the paragraph in all. Answers are found onPractice 18.

Many people think it is alright to except charitable assistance irregardless of their ability to secure financial indepandance for themselves. Other people think it can not effect them, or that they do not loose money, if these proffesional charity cases force the government to acomodate they’re free reign. I would not be exxagerating if I said that noone lives in a vaccum, and all descent people need to be responsable for there own welfare to. I wander if it would of ever ocured too these folks to ascent to by food, clothing and shelter firsthand for people who will not altar their indiscrete financial habits. I think its unlikely. When your looking passed the question of whose paying for these goods and services, than your excepting there right to steel from you’re pocket everyday.

Doubling Consonants. One of the most confusing issues in spelling is whether to double the final consonant when adding a suffix. Beginning or beginingTraveling or travelling? Here’s the rule: If the final syllable is accented and the final vowel is short, the consonant is doubled. (Except for c, because c before e or i is softened to an s sound; we simply add a k instead: e.g. picnic, picnicked.)

9781419552175_Interior_0059_005

9781419552175_Interior_0059_006

There are complications, of course. In American English, this rule only applies if the final syllable of the root word is accented; in British English, it applies whether or not the accent falls on the final syllable. Therefore, Americans abroad are travelers, while peripatetic Brits are travellers.

Another wrinkle involves words where the vowel-consonant pattern would normally indicate a long vowel, but the pronunciation is short, as in head or come. In this case, if we add a suffix, there is no need to protect the short vowel by doubling the consonant: heading rhymes with wedding, not with reading, while coming rhymes with humming, not with homing.

There are of course anomalies and exceptions, but following these guidelines will help you minimize your errors.

Practice 19

Find the mistake(s) in the following sentences. Answers are found on Practice 19.

1. In fields such as human developement and family plan-ing, it is becomming easier to find funding for basic research.

2. As the hailstorm intensified, the campers paniced and ran screamming through the grounds.

3. Effective leadership means recognizing when an objective cannot be reached without sacrificing essential resources.

Homonyms and Other Commonly Confused Words

Accept or exceptAlter or altarDiscrete or discreet? Even if you know the difference between these words, when you’re under time pressure, it’s easy to type in the wrong one. Homonyms and contractions (e.g., who’s) are especially good candidates for typos, so here’s a quick review of some of the most common troublemakers.

Note: Since we reviewed contractions vs. possessives already, they aren’t included in the list below. Remember that you can avoid contraction typos by simply avoiding contractions throughout your essay. (Besides, while the ETS E-rater is not programmed to grade down for the use of contractions, many professors (the sort of people who will be grading your essay!) feel strongly that contractions are not appropriate in academic prose.)

accept (v.): to take or receive. The CEO accepted the treasurer’s resignation.

except (prep.): leave out. The Town Council approved all elements of the proposalexcept the tax increase.

adverse (adj.): unfavorable. This plan would have an adverse impact on the environment.

averse (adj.): opposed or reluctant. I am averse to doing business with companies thatdon’t treat their employees fairly.

advice (n.): recommendation as to what should be done. I would like your advice abouthow to handle this situation.

advise (v.): to recommend what should be done. I will be happy to advise you.

affect (v.): to have an impact or influence on. The expansion of Pyramid Shopping Mall will certainly affect traffic on the access roads.

effect (n.): result, impact. The proposal will have a deleterious effect on everyone’squality of life. (v.): to cause, implement. The engineers were able to effect achange in train’s performance at high speeds.

altar (n.): an elevated structure, typically intended for the performance of religious rituals. The court refused to allow the construction of an altar on public property.

alter (v.): to change. It should be a simple matter to alter one’s will.

among (prep): used to compare three or more items or entities. We can choose fromamong dozens of styles.

between (prep): used to compare two items or entities. We can choose between thesetwo styles.

amoral (adj.): neither moral nor immoral; without a sense of moral judgment. Clairewas upset with the amoral discussion of terrorist acts.

immoral (adj.): morally wrong. Whatever beliefs a terrorist has, terrorist acts areimmoral.

ascent (n.): climb, upward movement. Vanessa’s rapid ascent up the corporate ladderimpressed us all.

assent (n.): agreement; (v.): to agree. Peter has given his assent to the plan.

assure (v.): to convince or guarantee. He has assured me that this is a safe investment.

ensure (v.): to make certain. Please ensure that this is a safe investment.

insure (v.): to guard against loss. There is no way to insure this investment.

bazaar (n.): traditional oriental market. I found these fantastic trinkets at the bazaar.

bizarre (adj.): very strange, weird. No one knew how to respond to such a bizarrequestion.

cite (v.): to quote, to refer. The article cited our annual report.

sight (n.): something seen or visible; the faculty of seeing. What an amazing sight!

site (n.): location; (v.): to place or locate. This is the perfect site for a new office.

complement (n.): something that completes; (v.): to go with or complete. This itemreally complements our product line.

compliment (v.): to flatter; (n.): a flattering remark. That was a sincere compliment.

continual (adj.): repeated regularly and frequently. Alan’s continual telephone calls finallywore Rosa down and she agreed to a meeting.

continuous (adj.): extended or prolonged without interruption. The continuous bangingfrom the construction site gave me a severe headache.

decent (adj.): proper, acceptable. You can trust Lena to do what is decent.

descent (n.): downward movement. The rapid descent of the balloon frightened its riders.

discrete (adj.): separate, not connected. These are two discrete issues.

discreet (adj.): prudent, modest, having discretion; not allowing others to notice. I must bevery discreet about looking for a new job while I am still employed here.

disinterested (adj.): impartial, objective. We need a disinterested person to act as anarbitrator in this dispute.

uninterested (adj.): not interested. Charles is uninterested, but he’ll come along anyway.

eminent (adj.): outstanding, distinguished. The eminent Judge Blackwell will teach aspecial seminar in business ethics this fall.

imminent (adj.): about to happen, impending. Warned of imminent layoffs, Lorettabegan looking for another job.

incidence (uncountable noun: occurrence) frequency

incident(pl.: incidents) (countable noun: events, cases) an occurrence of an event or situation

personal (adj.): private or pertaining to the individual. Please mark the envelope “personaland confidential.”

personnel (n.): employees. This year we had a 5% increase in personnel.

precede (v.): to come before. The list of resources should precede the financial worksheet.

proceed (v.): to go forward. Although Jules will be absent, we will proceed with themeeting as planned.

principal (n.): head of a school or organization, primary participant, main sum of money; (adj.): main, foremost, most important. Joshua is one of the principalsof the company.

principle (n.): a basic truth or law. I have always run my business based on the principlethat honesty is the best policy, even in a capitalist society.

rein (n.): a means of restraint or guidance; (v.) to restrain, control. You need to rein inyour intern, Carol—she’s taking on much too much responsibility and doesn’tseem to know what she’s doing.

reign(v.): to exercise power; (n.): period in which a ruler exercised power or a condition prevailed. Under the reign of King Richard, order was restored.

than (conj.): used to compare. I will be more successful this time because I am moreexperienced than before.

then (adv.): at that time, therefore. I was very naïve back then.

weather (n.): climatic conditions, state of the atmosphere. The bad weather is going tokeep people away from our grand opening.

whether (conj.): used to refer to a choice between alternatives. I am not sure whether Iwill attend the grand opening or not.

Practice 20

Find the mistake(s) in the following sentences. Answers are found on Practice 20.

1. Its often necessary to by more goods then can be consumed in a single day even though there likely to spoil.

2. I would of thought that journalists should avoid taking sides when they right about such controversial subjects.

3. We all here stories of people who’s children refuse to accompany them on vacations.

4. Can you site three incidences in the passed in which Congress has given its ascent to such a proposal?

5. The Board of Education excepted the advise of the high school principle in chosing to altar the wording of the teachers’ contracts.

6. After years of suffering under the heal of an autocrat, the citizens must steal themselves for change.

MAXIM 2: BE CLEAR

Correctness is important, but it means little if your sentences are not clear. Ambiguous, vague, and just plain confusing sentences can result from poor sentence structure or ineffective word choice.

Use Straightforward Sentence Structure

Some writers try to impress readers by writing elaborate sentences—but those sentences can often get tangled. Since the E-rater won’t be able to untangle your ideas, and since clarity is essential, it’s best to stick to straight-forward sentence structure: subject, verb,indirect object, object.

But that doesn’t mean you can’t write sophisticated sentences with lots of phrases and clauses. But you will need to be especially careful about where you put modifying clauses and phrases. The basic rule: Make sure your modifiers are as close as possible to the words they modify.

Modifier Placement

In English, the position of the word within a sentence often establishes the word’s relationship to other words in the sentence. This is especially true with modifying phrases. Like pronouns, adjectival expressions are generally connected to the nearest word that agrees with the modifier in person and number. Likewise, when a sentence contains more than one verb or verbal element (such as an infinitive, gerund, or participial), an adverbial expression will be interpreted as modifying the closest verb. The placement of prepositional phrases is particularly important, since they can modify both nouns and verbs as well as other elements.

AMBIGUOUS: The President and his closest advisors frequently discuss potential scandals behind closed doors [Which generally occur behind closed doors—the discussions or the scandals?]

CLEAR: Behind closed doors, the President and his closest advisors frequently discuss potential scandals.

CLEAR: The President and his closest advisors frequently discuss potential scandals that are occurring behind closed doors.

AMBIGUOUS: A politician must consider what compensation is expected for each campaign donation at the time it is given.
[Does at the time it is given modify consider or expected?]

CLEAR: When accepting a campaign donation, a politician must consider what compensation is expected.

CLEAR: When a politician receives a campaign donation, he or she must consider what compensation a donor expects in return.

Many adverbial expressions can refer to words that either precede or follow them. Ambiguity can result when the modifier is squeezed between two possible referents and the reader has no way to know which is the intended referent:

AMBIGUOUS: The pharmaceutical company announced suddenly researchers had succeeded in identifying the neurotoxin.
[Which was suddenthe company’s announcement or theresearchers’ success?]

CLEAR: Suddenly, the pharmaceutical company announced that researchers had succeeded in identifying the neurotoxin.

CLEAR: The pharmaceutical company announced that researchers had suddenly succeeded in identifying the neurotoxin.
[Notice how the use of that before the noun clause makes the sentence easier to understand.]

Avoid Vague or Ambiguous Language

Vague words are unclear; ambiguous words are words that have more than one possible meaning.

Vague Words and Phrases

Words like lots, somewhat, and really are vague. How much exactly? Your sentences will be clearer and more powerful if you use words that are more precise:

VAGUE: We saw lots of improvement in employee morale over the last six months.

MORE EXACT: We saw significant improvement in employee morale over the last six months.

VAGUE: Our choices for subcontractors are somewhat limited.

MORE EXACT: Our choices for subcontractors are severely limited.

Unclear and Ambiguous Pronoun References

A pronoun is a word that stands in for a noun (or noun expression) in a sentence. A pronoun must agree with its antecedent and must refer clearly to one and only one antecedent.

AMBIGUOUS: No entrepreneur should tell a client that he is overly concerned with image.
[Does he refer to entrepreneur or client?]

CLEAR: No entrepreneur should accuse a client of being overly concerned with image.

CLEAR: Being overly concerned with image is not something that an entrepreneur should admit to a client. or No entrepreneur should admit being overly concerned with image to a client.

Occasionally, you may have to repeat a noun, rather than rely on a pronoun that may make your sentence ambiguous.

AMBIGUOUS: It may be cost effective to rely on subcontractors instead of company personnel, as they would certainly require extra training.
[Who would require training—the subcontractors or the company personnel?]

CLEAR: It may be cost effective to rely on subcontractors instead of company personnel, as the company personnel would certainly require extra training.

An antecedent must actually occur in your text. Even if you think the reader will know what you mean, do not use a pronoun without a clear and appropriate antecedent.

INCORRECT: When you are voting for a candidate, you must be sure he or she is fully qualified to undertake it.
[What does it refer to? The antecedent is implied but must be clearly stated.]

CORRECT: When you are voting, you must be sure the candidate you choose is fully qualified to undertake the position in question.

Avoid using this, that, it, or which to refer to a whole phrase, sentence, or idea. Even when these pronouns are placed very close to their intended antecedents, the references may still be unclear.

UNCLEAR: U.S. consumers use increasingly large amounts of non-recyclable diapers every year. Some worry that this will someday turn the Earth into a giant trash can. [What exactly is this? The use of diapers or the trash that results from their use?]

CLEAR: U.S. consumers use increasingly large amounts of non-recyclable diapers every year. Some worry that this ever-growing mass of waste products will someday turn the Earth into a giant trash can.

UNCLEAR: The candidate changed his position on all the key issues, which made the voters extremely nervous.
[What makes the voters nervous—that the candidate changed position or the issues? Which could refer to either.]

CLEAR: The candidate changed his position on all the key issues, making the voters extremely nervous.

Ambiguous repetition generally involves pronouns or other words which may refer to multiple entities or concepts:

UNACCEPTABLE: They concluded that most teachers do like their students, and, while they may not clearly express their feelings, they generally do reciprocate this sympathy.

ACCEPTABLE: The researchers concluded that most teachers do like their students, and, while the children may not clearly express their feelings, they generally reciprocate their teacher’s sympathy.

UNACCEPTABLE: It is well known that that problem has been the subject of a full-fledged investigation, and that will most likely yield some temporary solution that will prove acceptable to all concerned parties.

ACCEPTABLE: It is well known that the problem has been the subject of a full-fledged investigation, which will most likely yield a temporary solution acceptable to all concerned parties.

MAXIM 3: BE CONCISE

Why take 200 words to express an idea that can be conveyed in 100? Unfortunately, some of us have been “trained” to use more words than necessary because we were often under pressure to write essays of a certain length. But 100 clear and concise words are much better than 200 words of fluff. And the way to move from 100 to 200 words if so required isn’t through “filler” words and phrases—it’s through the development of ideas.

Unnecessary words and phrases don’t improve writing; they bog it down and often irritate readers. Concise writing is clear writing; it avoids the clutter and confusion that often result from unnecessary wordiness.

Wordy Phrases

In an attempt to make their prose seem more scholarly or more formal, some test takers use phrases where single words will do: at the present time or at this point in time instead of now, or take into consideration instead of simply consider. Don’t. Instead, use the simpler, clearer phrase. Here are some more examples of wordy phrases and their concise counterparts:

WORDY

CONCISE

along the lines of

like

as a matter of fact

in fact

at all times

always

by means of

by

because of the fact that

because

by virtue of the fact that

because

due to the fact that

because

for the reason that

because

in light of the fact that

because

in this day and age

today

in order to

to

in spite of the fact that

although, though

in the event that

if

until such a time as

until

Unnecessary Relative Clauses

Wordiness is also caused by unnecessary thatwho, and which clauses and phrases. To be more concise and precise, remove the relative pronoun and verb to create an appositive or turn the phrases and clauses into adjectives:

WORDY: The Truman Doctrine, which was established in 1947, created a “policy of containment.”

CONCISE: Established in 1947, the Truman Doctrine created a “policy of containment.”

MORE CONCISE: The 1947 Truman Doctrine created a “policy of containment.”

WORDY: Temporary employees who work hard and are loyal are often rewarded with full-time employment.

CONCISE: Hard-working, loyal temporary employees are often rewarded with full-time employment.

Cluttered Constructions

There is, it is, and that often unnecessarily clutter sentences. A simple deletion or reconstruction can eliminate wordiness and create smoother sentences.

WORDY: It is essential that all visitors and employees wear safety goggles on the production floor.

CONCISE: All visitors and employees must wear safety goggles on the production floor.

WORDY: There is another reason that we should switch vendors, and that is that our current vendor does not offer bulk discounts.

CONCISE: We should also switch vendors because our current supplier does not offer bulk discounts.

Unnecessary Repetition

It is redundant to speak of “a beginner lacking experience’’: the word beginner in itself implies lack of experience. Yet we sometimes use repetitive phrases, especially when we are unsure that we are expressing our idea clearly or with sufficient force.

REDUNDANT

CONCISE

refer back

refer

few in number

few

small-sized

small

grouped together

grouped

in my own personal opinion

in my opinion

end result

result

serious crisis

crisis

new initiatives

initiatives

climb up

climb

Sometime a sentence unnecessarily “defines” a word again in an attempt to make an idea more clear:

WORDY: Jared lost a lot of business because of inflation, which drove up prices. [Inflation means an increase in prices, so the clause whichdrove up prices is entirely unnecessary.]

CONCISE: Jared lost a lot of business because of inflation.

WORDY: He is sad and depressed because he did not get promoted to a better position. [Sad and depressed are synonyms; promoted means to move up to a better position.]

CONCISE: He is depressed because he did not get the promotion.

MAXIM 4: BE EXACT

The more precise your language, the more impact your words will have. You can avoid wordiness and add power to your writing by using exact words and phrases.

IMPRECISE: Paris is a very beautiful city.

PRECISE: Paris is a stunning city.

Stunning is more precise, more concise, and more sophisticated than very beautiful—and thus a better word choice.

IMPRECISE: After a while, the odor disappeared into the air.

PRECISE: After a while, the odor dissipated.

IMPRECISE: The school had to close because it had many problems with safety.

PRECISE: The school was shut down because of many safety violations.
[Violations is more specific and concrete than problems withshutdown is more exact and powerful than had to close.

MAXIM 5: BE APPROPRIATE

For the Analytical Writing tasks, being appropriate means writing at the appropriate level of formality. You need a style, tone, and point of view that show respect for your readers—their authority, their level of intelligence, and their time.

Of course, you know your real audience consists of a trained professor and the E-rater. But you will be expected to write for a general audience of your peers—people with a general college-level education and varied interests and backgrounds. Which is a good thing, because thinking of your audience in this way can help you write more effectively and be more relaxed during the exam.

Avoid Slang and Colloquial Expressions

There are many levels of diction, and each genre has a range of dictions generally considered appropriate. Your essays do not require the most formal diction, but an overly colloquial style will drag down your score, and slang of any kind should be strictly avoided. The E-rater won’t know, for example, if you are using slang in dialogue to realistically represent how someone speaks; it will simply assume you are using inappropriate vocabulary. So remember: no slang. Zero. Zip. Zilch. Nada.

Colloquial language isn’t exactly slang, but it is informal and for the most part should also be avoided. Here are some particular expressions to avoid, as well as some acceptable substitutes.

Intensifiers

Awfully, incredibly, pretty, really, totally: all these words have legitimate uses, but they should not be used as generic intensifiers. Even very has been so over-used that it tends to diminish rather than emphasize your point. Preferable alternatives:

9781419552175_Interior_0070_006

Down-toners

Kind of, sort of, pretty much—again, these expressions have legitimate uses, but not in a formal essay. Besides being too colloquial, these terms are also vague.

Instead use words such as fairly, partially, largely, mildly, moderately, quite, rather, slightly,somewhat.

Quantifiers

Instead of lots, a lot of, a bunch of and other colloquial (and vague) quantifiers, try many, much, a number of, a substantial number of, quite a few, numerous.

Contractions

Coulda, gonna, hafta, hadta, lotsa, oughta, shoulda, wanna, woulda: we use these all the time in speech, but these colloquial contractions are not acceptable in formal writing.

Use Sophisticated Vocabulary

An appropriate style for the GRE and GMAT essays requires a formal and somewhat sophisticated level of vocabulary. Here are a few simple substitutes to help you write on a more sophisticated level.

71 Instead of like, use such as:

DEPRECATED: The success of a business depends on many factors, like convenient location, well-trained staff, and effective promotion.

ACCEPTABLE: The success of a business depends on many factors, such as convenient location, well-trained staff, and effective promotion.

71 Instead of different, use various.

71 Instead of big, use large, important, substantial, prestigious, significant.

71 Instead of okay, use acceptable, satisfactory, appropriate.

71 Instead of etc., use and so on (better yet, restructure the list and use including or forexample or include more examples).

71 Instead of oftentimes, use often.

UNSOPHISTICATED: For different reasons, I was turned down by a lot of big schools, like Harvard, Yale, Princeton, Dartmouth, etc.

SOPHISTICATED: For various reasons, I was rejected by many prestigious schools, including Harvard, Yale, Princeton, and Dartmouth.

Practice 21

Find the mistake(s) in the following sentences. Answers are found on Practice 21.

1. Lots of people are incredibly into video games and watching the tube, and I basically feel that that’s okay, although it’s pretty much of a waste of time.

2. These days there’re a lot of different reality shows, like Survivor, American Idol, etc., that are all kind of similar.

3. Oftentimes a perfectly good plan can be trashed by a minor oversight.

Avoid Jargon and Pretentious Language

Sophisticated does not mean pretentious. Pretentious texts try to impress by sounding scholarly or profound, often using over-the-top or esoteric vocabulary. You can be sophisticated without clouding meaning or annoying readers with pretentious language.

PRETENTIOUS: Avoid periphrasis and pleonasms in your compositions.

JUST RIGHT: Avoid wordiness and redundancy in your writing.

Jargon is technical or specialized language. Remember the rule: respect your reader. Not all of your readers will be familiar with specialized terms, and even if you think most of your readers are, that still leaves a few who will be confused by your word choice. So, either avoid jargon altogether and use lay terms or define any specialized language you use.

Practice 22

The following paragraph contains several examples of slang and substandard vocabulary. Rewrite the paragraph in a style more appropriate for the GRE or GMAT. Answers are found on Practice 22.

It is really pretty supportable to argue that studying an academic discipline changes your view of the world. Like say if you study history, you oftentimes read about a bunch of different facts that are equally as important as the ones you learned about in school, but that totally weren’t covered. Lots of big social trends and beliefs that we think shoulda been okay at this point in time turn out to be incredibly bogus. Studying a discipline like history can really change how we look at the world.

72

Use an Appropriate Point of View

Though some professors still preach “Don’t use I,” both the E-rater and the human reader are expecting you to express your opinion. So use the first person point of view, but not excessively—you don’t need I in every sentence. Just write assertively (see Maxim 7). Use your personal experiences as examples throughout, but don’t overdo it.

You can also address your readers as you (second person point of view), but don’t address them directly (e.g., dear reader), and don’t attribute words, ideas, or experiences to your readers. Your reader may not have seen or might not believe or might not know. Play it safe by using the generic third person (but not one, which often comes off sounding pretentious):.

WRONG: As you know…

WRONG: As one knows…

RIGHT: As many people know…

WRONG: You have undoubtedly seen… RIGHT: As you may have seen…

MAXIM 6: BE CONSISTENT

While variety is important in some aspects of writing (see Maxim 8), consistency is an important element your readers will be looking for. Consistency in writing shows evidence of control over your thoughts as well as the conventions of writing. Make sure your essay is consistent in style and tone as well as point of view.

Consistent Style

Style is created primarily through word choice and sentence structure. Do you tend to write in long, descriptive sentences? In a short, bare-bones kind of style? Do you prefer words like perambulate and hirsute to walk around and hairy?

You know that the most appropriate style for the essay is formal but not stiff or pretentious (see Maxim 5). If you tend to write in a very informal style, fancy up a bit for the Analytical Writing task. Don’t try to sound like someone else—stick to your natural voice—but choose words and sentence structures that are more sophisticated than normal. Whatever your natural style, start with an appropriate style and stick to it throughout your essay. A shift in style suggests an inconsistent grasp of the material, a lack of confidence in your own writing skills, or a gap in logic—all of which can be quite disconcerting to your readers.

INCONSISTENT: Lots of things can work together to create success, including education, smarts, foresight, perseverance, and sometimes plain old dumb luck.

CONSISTENT: Many factors contribute to success, including education, intelligence, foresight, perseverance, and sometimes serendipity.

INCONSISTENT: The reef is a vast ecosystem in the sea where a bunch of species live together. Lots of them are mutually dependent upon each other.

CONSISTENT: The reef is a vast ecosystem in the sea where hundreds of species live in symbiotic relationships.

Consistent Tone

Tone is the mood or attitude conveyed by writing or speech. In text, we convey tone through word choice, sentence structure, and punctuation. For example, both of the sentences below say the same thing, but the word choice and punctuation convey a different attitude towards the senator’s action.

The senator evaded another question.

The senator dodged another question!

You might have a great sense of humor, but these essays are not the place for a lighthearted or jocular essay. The most appropriate tone is one that is serious, informative, and always respectful of readers and other points of view. Start with an appropriate tone and stick to it throughout the essay.

Consistent Point of View

It is often useful to explore contrasting points of view within your essay—in fact, consideration of other opinions is one of the hallmarks of an effective argument. But your point of view as author should remain consistent within your essay. It is easy, for example, to slide around from I to you to one to we. The effect can be disconcerting, to say the least. One option to avoid is the use of one: it is grammatically limiting and it sounds pretentious. Whatever narrative voice you choose, and whichever voices you introduce to represent other perspectives, be consistent (but avoid overuse of particular pronouns).

TANGLED: I often feel that one has to make sacrifices to reach the goals you set for yourself, but we should always remember that life occurs in the present tense.

STRAIGHTFORWARD: I often feel that I must make sacrifices to reach my goal, but I try to remember that life occurs in the present tense.

STRAIGHTFORWARD: We may feel it necessary to make sacrifices in order to reach our goals, but we should always remember that life occurs in the present tense.

STRAIGHTFORWARD: Some people feel it necessary to make sacrifices in order to reach their goals, but they should always remember that life occurs in the present tense.

STRAIGHTFORWARD: You may feel it necessary to make sacrifices in order to reach your goals, but you should always remember that life occurs in the present tense.

MAXIM 7: BE ASSERTIVE

An assertive essay expresses confidence in its ideas, and this makes the essay far more convincing than one that is hesitant. Remember that these essays are designed in part to measure what you think and why you think it—so state your points without hesitation.

HESITANT: In my humble opinion, there are few things more important to leading a relatively successful life than health and companionship.

ASSERTIVE: Health and companionship are essential.

HESITANT: I think maybe we could come up with some way to make chief executives more accountable for what they do.

ASSERTIVE: Chief executives should be more accountable for their actions.

The more assertive sentences convey confidence, which goes a long way towards convincing readers of a particular point of view. They are also clearer and more concise than the hesitant versions.

On the other hand, there is no need to bludgeon the graders with your certitude. A few qualifiers will give the impression that you are reasonable: fairly, rather, somewhat, relatively, and of such expressions as seems to be, a little,and a certain amount of are good choices. But, do not overplay the modesty card.

TOO ASSERTIVE: In light of the facts, it is absolutely certain that the election scandal could have been avoided.

TOO MODEST: In light of the facts, it seems fairly reasonable that perhaps the election scandal might have been avoided. [Too wordy as well.]

JUST RIGHT: In light of the facts, it seems certain that the election scandal could have been avoided.

Avoid Needless Self-Reference

Like letters to the editor of a newspaper, Analytical Writing essays are by definition expressions of your views; it is unnecessary to label every assertion as opinion. Two or three self-references may be appropriate, especially when distinguishing your point of view from another, but there is no need to draw attention to yourself. Keep your focus on the topic.

OVERLY SELF-REFERENT: I used to believe that we should have a choice on matters of personal safety such as whether or not to wear a helmet when riding a motorcycle, but I now realize that personal safety is actually a matter of public welfare.

JUST RIGHT: Personal safety is a matter of public welfare.

BEST FOR AN OPENING PARAGRAPH: I believe personal safety is a matter of public welfare.

Use the Active Voice

Let’s be clear: the passive voice is an extremely useful device, particularly for expressing ideas where the speaker does not know the agent or does not wish to reveal it:

EXAMPLE: Recently, two schools in my district were vandalized.

The passive voice is also useful in manipulating sentence balance, particularly when the subject is long and involved and the predicate is brief.

AWKWARD: Often, newspapers that are having problems with falling circulation, rising costs, and the threat of a hostile buy-out from an international publishing conglomerate hire media consultants.

BETTER: Often, media consultants are hired by newspapers that are having problems with falling circulation, rising costs, and the threat of a hostile buyout from an international publishing conglomerate.

However...the passive voice has been notoriously overused by weak writers trying to sound authoritative, to the point that graders (human as well as computerized) generally react negatively to it. Solution: wherever possible, use the active voice. Active sentences are more direct and more concise, conveying your ideas with more clarity and power.

PASSIVE: The active voice should be used by essay writers.

ACTIVE: Essay writers should use the active voice.

State Your Opinions

Our research indicates that the E-rater is looking for expressions of opinion in the first paragraph: I believein my opinionI think. However, it is also looking for less obvious (but no less direct) expressions of judgment: assume, exaggerate, fail, ignore, misconstrue,misjudge, misinterpret, misrepresent, overemphasize, overestimate, overlook, presume,underestimate, understate, and so on. Words that indicate consideration of counterarguments or alternative points of view are also regarded as signs of engaged analysis. These expressions include actually, admittedly, although, despite, except, even though,however, in spite of, nevertheless, nonetheless, notwithstanding, still, and yet.

Practice 23

The following sentences use passive voice unnecessarily. Rewrite them in active voice. Answers are found on Practice 23.

1. The coolant pumps were destroyed by a surge of power.

2. The transformer was struck by a bolt of lightning.

3. The goalkeeper was too slow to stop the ball.

4. The moods of a manic-depressive are unpredictable.

5. The administrative secretary is responsible for monitoring and balancing the budgets.

6. It is important that hikers remain on marked trails.

7. All too often, athletes with marginal academic skills have been recruited by our coaches.

8. Teachers have been portrayed or stereotyped as illiterate, even though they fulfill strict requirements.

9781419552175_Interior_0077_011

MAXIM 8: BE EXCITING

Whatever you actually say in your essay, how you say it can have a significant impact on your score. Your essay can be correct, concise, appropriate—all of the other maxims—yet still be dull and dry and at risk for a lower score. A lively and engaging essay will earn more points from both your human and computer graders. You can make your essay inviting to readers by having variety in both your sentence structure and vocabulary.

Variety in Sentence Structure

Human and mechanical graders are looking for a variety of sentence forms. What that means, specifically, is that you’ll want to use a combination of simple, compound, complex, and compound-complex sentences. Here are some examples:

Simple (one main clause):
Any entrepreneur seeking a new business location should seriously consider
Nashville, Tennessee.

Compound (two or more main clauses):
Entrepreneurs often need to select a new business location, and they should seriously consider Nashville, Tennessee.

Complex (one main clause, one subordinate clause):
If any entrepreneurs are looking for a new business location, they should seriously consider Nashville, Tennessee.

Compound-complex (two or more main clauses, plus at least one subordinate clause):
Entrepreneurs often need to select a new business location, and when they do, they should seriously consider Nashville, Tennessee.

In addition to using a variety of these basic sentence forms, you can enliven your sentences by placing “interrupters,” phrases and clauses in various places:

X is unlike Y because of Z.
Because of Z, X is unlike Y.
However, X is unlike Y because of Z.
X, however, is unlike Y because of Z.
However, because of Z, X is unlike Y.

Practice 24

Fill in the table below. Each sentence is provided in either simple, compound, complex, or compound-complex form. Rewrite each given example in the three other forms. Answers are found on Practice 24.

Simple

Sometimes the fresh perspective of a non-expert can be

 

valuable in the consideration of a subject.

Compound

 
   
   

Complex

 
   
   

Compound-Complex

 
   
   

Simple

 
   
   

Compound

The feeling of having fulfilled a personal goal is important, but

 

the tangible rewards of society are at least as important.

Complex

 
   
   

Compound-Complex

 
   
   

Simple

 
   
   

Compound

 
   
   

Complex

Even though Company B is more expensive, the important

 

question is whether the combined cost of pest-control and

 

savings in product damage are greater with Company B.

Compound-Complex

 
   
   

Simple

 
   
   

Compound

 
   
   

Complex

 
   
   

Compound-Complex

Different academic communities have different traditions, and

 

while these differences may be significant, it is an oversimplification

 

to say that there can be no meaningful interaction between

 

them

Simple

It is reasonable to argue that a cheaper brand of sunscreen

 

might encourage people to have a false sense of security.

Compound

 
   
   

Complex

 
   
   

Compound-Complex

 
   
   

Length

The scorers are also looking for a variety of sentence lengths. A short sentence (four to eight words) can effectively emphasize a simple point; a long sentence (30–45 words) might be necessary to present a relatively complicated idea. Varying sentence length throughout also creates a more pleasing rhythm for your readers—too many short sentences are likely to sound monotonous while too many long sentences may be difficult on your reader.

EXAMPLE: Entrepreneurs often need to select a new business location, and when they do, they should seriously consider Nashville, Tennessee. This southern city has a great deal to offer.

Sentence Openers

How you start your sentences should vary, too. If all of your sentences start with the subject, even if the sentence lengths and forms vary, it can sound awfully monotonous (unless you are purposely using parallel construction).

MONOTONOUS: Nashville was founded in 1779. Tenessee became the sixteenth state of the union in 1796. Nashville became the state’s capital in 1812.

EXCITING: Founded in 1779, Nashville became the capital of Tennessee in 1812, sixteen years after Tennessee became the sixteenth state of the union.

To add variety, combine sentences as in the example above and start some sentences with introductory clauses and phrases rather than the subject. But keep the basic order for sentence structure for your core clause (subject, verb, indirect object, object).

NEEDS VARIETY: Nashville was founded in 1779. It became the state capital in 1812.

HAS VARIETY: Founded in 1779, Nashville became the state capital in 1812.

A Varied Vocabulary

Repetition is one of our most effective rhetorical devices. As we have already pointed out, the E-rater cannot measure the quality of essay content directly. Instead, it focuses on surface characteristics that ETS researchers believe are statistically symptomatic of good or bad writing. Not only does it look for and penalize excessive repetition, but the E-rater also compares the diversity of vocabulary of each submitted essay with prompt-specific vocabulary lists as well as essay-type vocabulary lists. Thus, one of our most important sentence-level strategies is to use synonyms instead if relying on a few key words throughout your essay.

REPETITIVE VOCABULARY: Business is not an occupation for the faint of heart. Every businessperson should be aware that nearly 50% of businesses fail in their first year, while 75% of businesses go under within three years. Our first order of business in this essay is to consider the question, “What are the causes of business failure?”

SYNONYM-ENRICHED: Business is not an occupation for the faint of heart. Every entrepreneur should be aware that nearly 50% of new ventures fail in their first year, while 75% go under within three years. Our first concernin this essay is to consider the question, “What are the causes of commercial failure?”

An effective way to avoid repetition is to make lists of terms pertinent to the various prompt topics. For example, here is a clustered list of nouns that might be useful in answering a question about higher education.

Bullet facilities, campus, library, dormitory, laboratory, location, faculty-to-student ratio, sports

Bullet recruitment, scholarship, financial aid, stipend, assistance, promotional material, catalogue, publication, publicity, public relations

Bullet admissions, requirements, standards, entrance, application

Bullet administration, administrator, dean, official, board of trustees, committee

Bullet faculty, professor, advisor, candidate, academic, researcher, lecturer, teaching assistant, educator

Bullet student, applicant, candidate, alumnus, registrant, freshman, sophomore, junior, senior, undergraduate, graduate, scholar, researcher

Bullet field, major, research, study, discipline, program

Bullet instruction, courses, curriculum, elective, required course, lecture, seminar, workshop, lab, syllabus, assigned readings, assignments, homework

Bullet grades, marks, scores, average, points, GPA

Bullet test, competition, examination, quiz, midterm, final

Bullet college, university, institution, higher education, academia, school, junior college,

Bullet goals, objectives, degree, graduation, commencement, employment, career, job, placement, opportunity

Practice 25

The following paragraph contains examples of overly repetitive word use. Correct the problems by rewriting the paragraph with appropriate synonyms in place of the repetitive words and phrases. Answers are found on Practice 25.

Courses that focus on intellectual development are more important than courses that contribute to professional development. Courses focused on professional development assume that these courses will still be relevant to the future job market, while in fact the constant changes in the job market might make such courses obsolete. In contrast, courses that work toward intellectual development are courses that train a person for a variety of job market roles, so that even if the job market changes, the work done in the courses remains relevant. This is not to say that many courses cannot do both: preparing a person for the job market while also preparing her for a variety of different job market possibilities. But, while courses on professional development have their place in the realm of university courses, they should not be allowed to supersede courses that train the intellect for a changing job market.

9781419552175_Interior_0083_004

EIGHT MAXIMS, REDUX

Here’s a quick recap of our Eight Maxims of Effective Writing:

Be correct.

Be clear.

Be concise.

Be exact.

Be appropriate.

Be consistent.

Be assertive.

Be exciting.

Check your answers to the practice exercises from this chapter on the following page. To learn about writing strong paragraphs and essays, turn to chapter four.

PRACTICE ANSWERS AND EXPLANATIONS

Practice 1

1. Most of us wish that our parents were better prepared to face retirement.

2. While we may wish that our physical conditioning were better, few of us are prepared to invest the time and effort in the kind of exercise and diet that might help us achieve it.

3. No mistake.

4. Although it is simple to mouth words of support for the work of others, it is preferable that they be given negative feedback where it applies.

5. No mistake.

6. Judging from recent pricing patterns, it is imperative that the American government begin to regulate retail milk prices

Practice 2

1. Many people in New York travel by subway. 
Many is a singular indefinite pronoun.

2. Workers in New York often commute a long way to work. 
The prepositional phrase in New York interferes with the connection between the plural subject, workers and the plural verb commute.

3. Tourists in New York expect to see the subway. 
The prepositional phrase in New York separates the plural subject tourists and the plural verb expect.

4. The “redbird” subway cars with a red body have been a common sight in New York until recently. 
The prepositional phrase with a red body might mislead you about the connection between the plural subject cars and the plural verb have been.

5. The streets of large cities such as New York are undercut by complex networks of subway tunnels. The long-distance phrases of large cities such as New York interferes with your perception of the relationship between the plural subject streets and the plural verb are.

6. Today, the economy of New York and other large cities is booming. 
The word cities next to the verb can interfere with your sense that the subject economy matches the singular verb is.

7. New Yorkers with a good income are less likely to commute by subway. 
The prepositional phrase with a good income might confuse the relationship between the plural subject New Yorkers and the plural verb are.

8. A worker with a long commute does not want to spend hours on the subway. 
The singular subject worker takes the singular verb form does.

9. A private car, although convenient, pollutes the air. 
The subject car is singular, so it takes the singular verb form pollutes.

10. Parking in one of New York’s many overcrowded garages is also a problem. 
The singular subject parking requires the singular verb is. Don’t be misled by the plural word garages because it is part of a prepositional phrase, not the subject of the sentence.

11. Every day, Joe and Carla ride the subway to work. 
A compound subject joined by and is plural.

12. Every day, Joe or Carla rides the subway to work. 
In a compound subject joined by or, the verb matches the closer part of the subject. Carla is singular, so use the singular verb rides.

13. Every day, Joe’s sisters or Carla’s sisters ride the subway to work.
In a compound subject joined by or , the verb matches the closer part of the subject. Carla’s sisters is plural, so use the plural verb ride.

14. Every day, Joe’s sisters or Carla rides the subway to work. 
In a compound subject joined by or, the verb matches the closer part of the subject. Carla is singular, so use the singular verb rides.

15. Every day, Carla or Joe’s sisters from Long Island ride the subway to work. 
In a compound subject joined by or, the verb matches the closer part of the subject. Don’t be thrown by the prepositional phrase from Long Island. Since Joe’s sisters is plural, use the plural verb ride.

16. Everyone enjoys a summer vacation. 
Most indefinite pronouns, including everyone, are singular.

17. Nobody has fun when the Cyclones lose a game. 
Most indefinite pronouns, including nobody, are singular.

18. Either of the answers is valid in response to that question. 
Either is a singular indefinite pronoun. Disregard the prepositional phrase of theanswers when figuring out the proper verb form.

19. Each of the students brings a book to class every day. 
Each is a singular indefinite pronoun. Disregard the prepositional phrase of the students when working out subject-verb agreement for this sentence.

20. Many are obsessed with reality television these days. 
Many is one of a very few plural indefinite pronouns in English.

Practice 3

1. The installation of video cameras in public areas certainly adds a measure of security but may eventually erode our right to privacy.

2. Competition for grades, jobs, and mates ultimately benefits society.

3. The main flaw in most of these arguments is the reliance upon unsupported inferences. or In most of these arguments, the main flaw is the reliance upon unsupported inferences.

Practice 4

1. The entire team of scientists was allergic to the very chemicals they were studying

2. No mistake.

3. A number of Internet companies are doubtless preparing to challenge Google for dominance of the search engine market.

Practice 5

1. The CEO, along with the Board of Directors, is responsible for any infraction of the corporation’s environmental protection policy.

2. Either the Attorney General or his senior assistants have the option of prosecuting such violations.

3. No mistake.

Practice 6

1. Looking at the data carefully, we see that the premises simply will not support the conclusion.

2. Having selected an appropriate brand name, the entrepreneur still faces many obstacles to successful marketing of the new product.

3. In this argument, an essential inference is that to pass the course one must take the exam.

Practice 7

1. For example, I would say that my roommate could be characterized as a poor student because he waited until the last minute to study for exams, wrote his lab reports without completing the assigned experiments, and lacked motivation.

2. It is reasonable for a Kravis Software sales representative to expect that he will have an opportunity to introduce his products at the meeting, that there will be a projector for his slide presentation, and that prospective buyers will ask questions about the product.

3. We would like to hire someone who is eager, responsible, and hardworking.

Practice 8

Sentence fragments are underlined in the paragraph below:

Everyone feels shy. At some point in life. It is perfectly normal to be concerned about how strangers might view you in an unfamiliar situation. For example, a social gathering or new job. However, shyness can sometimes become a major difficulty. If a person feels overpowering anxiety about common situations such as going to the store, attending classes at school or even walking down the street. That anxiety can interfere with the person’s ability to carry on a normal life. A few shy people develop a serious fear of strangers. Forcing them to restrict or avoid contact with people most of us interact with daily. Such as the mail carrier, co-workers or teachers. These people who feel overwhelmed by the mere thought of contact with a stranger. Such people may have agoraphobia.

One suggested way to fix the fragments in this paragraph is to connect sentences and drop words that make sentences into subordinate clauses:

Everyone feels shy at some point in life. It is perfectly normal to be concerned about how strangers might view you in an unfamiliar situation, for example, a social gathering or new job. However, shyness can sometimes become a major difficulty. If a person feels overpowering anxiety about common situations such as going to the store, attending classes at school or even walking down the street, that anxiety can interfere with the person’s ability to carry on a normal life. A few shy people develop a serious fear of strangers, forcing them to restrict or avoid contact with people most of us interact with daily, such as the mail carrier, co-workers or teachers. These people feel overwhelmed by the mere thought of contact with a stranger. Such people may have agoraphobia.

Practice 9

1. Elementary schools must impart the tools necessary to teach the basic skills. 
A restrictive element (“to teach the basic skills”) should not be set off by a comma.

2. The role of providing lifelong assistance to disabled people belongs to the government, which can muster the vast resources needed to properly care for the ill. 
The prepositional phrase “of providing…” is not parenthetic, so it doesn’t need commas. But the nonrestrictive phrase “which can muster…” does need to be set off by a comma. Nonessential phrases, which begin with the word “which,” are nonrestrictive.

3. Advocates of the proposed law, however, will most likely insist on the need to forestall improper sharing of intellectual property and classified information. 
The word “however,” in this case, is parenthetic, so set it off with commas.

4. All the support for this argument is either flawed, superfluous or irrelevant.
OR
All the support for this argument is either flawed, superfluous, or irrelevant. Items in a series should be separated by commas. Although current usage favors omitting the last comma just before the conjunction (“or” in this case), either method is correct.

5. In fact it is just as likely that some other cause can explain why the products at the uptown factory are cheaper to produce.
OR
In fact, it is just as likely that some other cause can explain why the products at the uptown factory are cheaper to produce. 
The introductory clause “in fact” is short and is unlikely to cause any confusion in this sentence. Although current usage favors omitting the comma that sets off a short introductory clause, either usage is correct.

6. Although discounted,
the evidence of the second survey is actually more informative to this argument.
In this case, the comma is required even though the introductory clause is short. Without the comma, the clause “Although discounted” is likely to cause confusion as the word “discounted” runs into the first part of the subject “the evidence,” that immediately follows it, so the comma is required to prevent confusion.

Practice 10

1. We may well ask ourselves what colleges can possibly do to prevent cheating.

2. No mistake.

3. For years, the largest contingents of international peacekeepers have come from the U.K. and the U.S.A.

Practice 11

1. When a society is in agreement on the need to meet social objectives, government generally takes on a powerful role; in such cases, taxation, rather than simply raising money, becomes, in addition, a means of implementing those goals.

2. The assets of such an enterprise might include, for example, $30 million in real estate, equipment and infrastructure; $20 million in cash, investments, and accounts receivable; and
$10 million in inventory.

3. No mistake.

Practice 12

1. No mistake.

2. No mistake.

3. No mistake.

Practice 13

1. Three principles are at issue in this case.

2. Ultimately, a leader must be guided by her people’s wishes.

3. With very few exceptions, women’s rights have been adjudicated by the courts over the past few decades.

4. Many Princes of Wales have served long terms as the heir to the crown of Great Britain, and Prince Charles is no exception.

Practice 14

1. In most developing countries, up to 80% of the population lives in rural communities.

2. No mistake.

3. Humans are not the only primates that have been known to murder their own kind: infanticide is well-documented among chimpanzeesbaboons, and several species of monkeys.

Practice 15

1. In the age of electronic media, reading books and magazines is not as important as it once was.

2. Consumers must have the confidence that they have an option to get a refund or to exchange any online purchase that does not meet with their satisfaction.

3. Statistics for births per year per capita can give a useful measure of a nation’s reproductive status.

Practice 16

Below is the corrected paragraph. Explanations are found on the following page.

(1) Abraham Lincoln described the U.S. government as being, as he said in the Gettysburg Address, “of the people, by the people, and for the people.” (2) This notion, I believe, puts the priorities between the achievements of institutions and the achievements of governments in proper perspective. (3) It is not a question of the achievements of individuals vs. the achievements of the government; that contrast is a false dilemma. (4) Rather, it is the achievements of individuals, which make up the achievements of a government, that determine the greatness of a nation. (5) This is what Lincoln meant when he referred to the nation’s “unfinished work”: that each of us has a task to perform, and those tasks determine what kind of nation we will live in. (6) With this fundamental argument in mind, how can we not join with Lincoln in wishing that such a nation “shall not perish from the earth?”

Explanations

(1) The expression as he said in the Gettysburg Address is a parenthetic, and so is enclosed in commas. The quotation beginning of the people is a direct quotation, so it is introduced by a comma. The three items in the quote are in a series, so they should be separated by commas. Note that the comma before the conjunction in the last item andfor the people is technically optional, although Lincoln included it in the written version of his speech so in this case it is probably best not to omit it. Also, omit the extra space between the initials of the abbreviation U.S.

(2) The phrase I believe is parenthetic, and so should be set off with commas. The phrase and the achievements of government is not parenthetic (and not in a series) so it should not be set off by commas. Omit the apostrophe on the simple plural priorities.

(3) This sentence is a run-on, having two independent clauses joined by a comma with no conjunction. Probably the best way to fix the run-on here is to replace the comma with a semicolon as shown (a period and new sentence would also work). Also, there’s an incorrect abbreviation (with an extra space) for versus. Finally, omit the slash from Analytical Writing essays.

(4) The comma after the short introductory rather is not absolutely vital, but probably contributes to avoiding misunderstanding in this case. The nonrestrictive clause whichmake up…government must be set off with commas.

(5) The “unfinished work” belongs to the nation, so nation’s takes a possessive apostrophe. The terminal colon goes outside the quotation marks. The part of the sentence after the colon consists of two independent clauses joined by a conjunction, so the clauses must be separated by a comma.

(6) The first part of this sentence, With this fundamental…mind, is a long introductory prepositional phrase, so set it off with a comma. The sentence itself is a question (beginning with how) so it must end with a question mark—which goes inside the quotation marks. Finally, the quote is run into the sentence, so it doesn’t take an introductory comma.

Practice 17

1. During the Middle Ages and the most of the Renaissance, philosophy was hardly distinct from theology.

2. By the end of springAmerican and Canadian university students have been enjoying their summer vacation for well over a month.

3. Widely regarded as the most important U.S. trade agreement in the past two decades, the U.S.-Canada Free Trade Agreement phased out all tariffs and quotas between the two countries by the end of the twentieth century.

4. Last year, Doctor Sanford was elected president of the Brooklyn Council. The president of the council meets regularly with the Dean of Students.

5. American Independence Day and Memorial Day occur during the summer, but Labor Day is in the fall.

6. The novels of William Faulkner are set in the American South, and the author himself lived most of his life south of the Mason-Dixon Line.

Practice 18

Many people think it is all right to accept charitable assistance regardless of their ability to secure financial independence for themselves. Other people think it cannot affect them, or that they do not lose money, if these professional charity cases force the government to accommodate their free rein. I would not be exaggerating if I said that no one lives in a vacuum, and all decent people need to be responsible for their own welfare too. I wonder if it would have ever occurred to these folks to assent to buy food, clothing and shelter firsthand for people who will not alter their indiscreet financial habits. I think it’s unlikely. When you’re looking past the question of who’s paying for these goods and services, then you’re accepting their right to steal from your pocket every day.

Practice 19

1. In fields such as human development and family planning, it is becoming easier to find funding for basic research.

2. As the hailstorm intensified, the campers panicked and ran screaming through the grounds.

3. No mistake.

Practice 20

1. It’s [or: It is] often necessary to buy more goods than can be consumed in a single day even though they’re [they are] likely to spoil.

2. I would have thought that journalists should avoid taking sides when they write about such controversial subjects.

3. We all hear stories of people whose children refuse to accompany them on vacations.

4. Can you cite three incidents in the past in which Congress has given its assent to such a proposal?

5. The Board of Education accepted the advice of the high school principal in choosing to alter the wording of the teachers’ contracts.

6. After years of suffering under the heel of an autocrat, the citizens must steel themselves for change.

Practice 21

1. Many people are obsessed with video games and televisionwhich I believe are relatively harmless wastes of time.

2. These days there are many reality shows, such as Survivor and American Idol, that are all essentially similar.

3. Often a perfectly good plan can be ruined by a minor oversight.

Practice 22

Your version may vary.

It is supportable to argue that studying an academic discipline changes your view of the world. For example, if you study history, you often read about many facts that are as important as the ones you learned about in school, but which weren’t covered. Many large social trends and beliefs that we think are valid today turn out to be wrong.

Studying a discipline such as history can change how we look at the world.

Practice 23

1. A surge of power destroyed the coolant pumps.

2. A bolt of lightning struck the transformer.

3. The slow goalkeeper failed to stop the ball.

4. Manic-depressives experience unpredictable moods.

5. The administrative secretary monitors and balances the budgets.

6. Hikers must remain on marked trails.

7. All too often, our coaches have recruited athletes with marginal academic skills.

8. The media portrays or stereotypes teachers as illiterate, even though they fulfill strict requirements.

Practice 24

Simple

Sometimes the fresh perspective of a non-expert can be

 

valuable in the consideration of a subject.

Compound

Experts often have the best advice about a subject, but some

 

times the fresh perspective of a non-expert can be valuable too.

Complex

Although experts often have the best advice about a subject,

 

sometimes the fresh perspective of a non-expert can be valuable too.

Compound-Complex

Because experts typically have the most information about a

 

subject, they often have the best advice about that subject, but

 

sometimes the fresh perspective of a non-expert can be valuable too.

Simple

The tangible rewards of society are at least as important as the

 

feeling of having fulfilled a personal goal.

Compound

The feeling of having fulfilled a personal goal is important, but

 

the tangible rewards of society are at least as important.

Complex

Although the feeling of having fulfilled a personal goal is important,

 

the tangible rewards of society are at least as important.

Compound-Complex

Rewards are important, and although the feeling of having fulfilled

 

a personal goal is important, the tangible rewards of society

 

are at least as important.

Simple

The important question is whether the combined cost of pestcontrol

 

and savings in product damage are greater with

 

Company B.

Compound

The important question is not which pest-control company is

 

cheaper, but rather which company provides the best combination

 

of cost and savings in product damage.

Complex

Even though Company B is more expensive, the important

 

question is whether the combined cost of pest-control and

 

savings in product damage are greater with Company B.

Compound-Complex

Even though Company B is more expensive, if the combined

 

cost of pest-control and savings due to product damage is

 

greater, then Company B is a better deal overall.

Simple

It is an oversimplification to say that there can be no meaningful

 

interaction between academic communities with different traditions.

Compound

Different academic communities have different traditions, but it

 

is an oversimplification to say that these differences preclude

 

meaningful interaction between them.

Complex

Even though different academic communities may have different

 

traditions, it is an oversimplification to say that there can be no

 

meaningful interaction between them.

Compound-Complex

Different academic communities have different traditions, and

 

while these differences may be significant, it is an oversimplification

 

to say that there can be no meaningful interaction between

 

them.

Simple

It is reasonable to argue that a cheaper brand of sunscreen

 

might encourage people to have a false sense of security.

Compound

The cheaper brand of sunscreen is less effective, and so the

 

sense of security it gives is false.

Complex

Because the cheaper brand of sunscreen is less effective, it is

 

certainly reasonable to say that it encourages a false sense of security.

Compound-Complex

Because the cheaper brand of sunscreen is less effective, and

 

because it encourages people to stay in the sun longer, it is fair

 

to say that it encourages a false sense of security.

Practice 25

Answers will vary slightly.

Repetitive words and phrases in this paragraph: courses, development, job market Synonyms for “courses”: lessons, classes, curricula, programs, learning, education Synonyms for “development”: improvement, training, education, advancement,enhancement

Synonyms for “job market”: employment, professional setting, occupation, trade, theworld of work

Sample rewrite using these synonyms:

A curriculum that focuses on intellectual improvement is more important than a curriculum that contributes to professional training. An educational program that emphasizes professional advancement assumes that this education will still be relevant to the future occupation of the student, while in fact the constant changes in the job market might make such learning obsolete. In contrast, a course of study that works toward intellectual growth trains a person for a variety of occupational roles, so that even if the demands of employers change, the educational program remains relevant. This is not to say that many curricula cannot do both: preparing a person for employment in a specific field while also preparing her for a variety of potential vocations. But, while classes that provide professional training have their place in the realm of university curricula, they should not be allowed to supersede education that trains the intellect for the ever-changing world of work.